You are on page 1of 65

Environment – 100

1. Which of the following is/are the 2. Consider the following statements with
components of Fly ash? reference to Fly Ash:
1. Silicon dioxide 1. National Thermal Power Corporation
(NTPC) is fulfilling its commitment
2. Aluminium oxide
towards 100% utilization of fly ash
3. Ferric oxide from power plants.
4. Calcium oxide 2. Fly ash when combined with water
Select the correct code: cause leaching of heavy metals in
ground water.
A. 1 and 2 only
Which of the statements given above is/are
B. 3 and 4 only
correct?
C. 4 only
A. 1 only
D. 1, 2, 3 and 4
B. 2 only
Answer: D
C. Both 1 and 2
Explanation:
D. Neither 1 nor 2
Fly Ash, also known as coal ash or pulverised
Answer: C
fuel ash is a coal combustion product
composed of fine particles of burned fuel. It is Explanation:
derived out of coal-fired boilers together with Statement 1 is correct: Recently, National
flue gases. Thermal Power Corporation (NTPC) has
In the past, fly ash was generally released into developed an infrastructure to transport fly
the atmosphere, but air pollution control ash from power plants in bulk to cement
standards now require that it be captured plants, at a cheaper cost. It will pave the way
prior to release by fitting pollution control for efficient and environment-friendly
equipment. transportation.
This development is in line with NTPC's
All the statements are correct.
commitment towards 100% utilization of fly
Fly ash particles are oxide rich and consist of ash from power plants. At present, 63% of the
silica, alumina, oxides of iron, calcium, and fly ash is being utilised in India.
magnesium and toxic heavy metals like lead, Statement 2 is correct: Fly ash when combined
arsenic, cobalt, and copper. with water cause leaching of heavy metals in
Major oxides are present are aluminium ground water.
silicate (in large amounts), silicon dioxide
Groundwater Contamination: Coal, which is
(SiO2) and calcium oxide (CaO).
a component of Fly Ash, contains elements
such as arsenic, barium, boron,
selenium and mercury, most of which is toxic
to human and animal life. Thus, fly ash
obtained after combustion of the coal has moth repellents, wood preservatives, and
potential to pollute groundwater. other products.
Ecological Impact: The Ecological effect of fly Health effects – irritation of the eye, nose and
ash will depend on the type of factory and throat, headaches, nausea and loss of
thermal power plant discharge at play. This is coordination.
due to the different chemical make-up of the Long term effects – suspected to damage the
coal based on the geological makeup of the liver and other parts of the body.
environment. Thus, the fly ash that will be
deposited on the topsoil will affect the plants Statement 1 is correct: Benzene is most
and animals in the surrounding area. harmful of all the VOCs. It is called a
carcinogen. It is commonly found in fuels,
cigarette smoke, cars, paints.
3. Consider the following statements with
Statement 2 is incorrect: The water solubility
reference to Volatile Organic Compounds
of volatile organic compounds is very high.
[VOCs]:
They are highly unstable at room
1. Benzene is most harmful of all the temperature.
VOCs. Statement 3 is correct: VOCs react with
2. The water solubility of volatile organic harmful pollutants and neutralise their effects
compounds is very high on the environment. 90% of the VOC in the
atmosphere is produced by the plants. The
3. VOCs react with harmful pollutants
VOCs play a major role in chemical processes
and neutralise their effects on the
that occur in the atmosphere. The VOCs keep
environment.
the atmosphere above it clean. VOCs are
Which of the statements given above is/are highly important for animal–plant
correct? communication.
A. 1 and 2 only
B. 2 and 3 only 4. Consider the following statements with
C. 1 and 3 only reference to Sulphur Dioxide:

D. 1, 2 and 3 1. Sulfur dioxide inhibits photosynthesis

Answer: C 2. It is abundantly available in the


atmosphere of Venus.
Explanation:
Volatile Organic Compounds (VOCs) are a 3. Production of paper is one of the
large group of carbon-based chemicals that sources of Sulphur Dioxide
easily evaporate at room temperature. Which of the statements given above is/are
Formaldehyde causes irritation to the eyes correct?
and nose and allergies. A. 1 and 2 only
The main indoor sources are perfumes, hair B. 2 and 3 only
sprays, furniture polish, glues, air fresheners,
C. 1 and 3 only
D. 1,2 and 3 Statement 3 is correct: Production of paper is
Answer: D one of the sources of Sulphur Dioxide

Explanation:
• It is a toxic gas with a pungent, 5. Montreal Protocol is associated with
which of the following?
irritating smell.
A. Ozone
• It contributes to acid rain.
B. Nitrous Oxide
• Sulfur dioxide inhibits photosynthesis
by disrupting the photosynthetic C. Chlorofluorocarbons
mechanism. D. Carbon Monoxide
• It is released naturally by volcanic Answer: A
activity.
Explanation:
• It is abundantly available in the
The Montreal Protocol on Substances that
atmosphere of Venus.
Deplete the Ozone Layer (a protocol to the
• Sulphur dioxide is primarily Vienna Convention for the Protection of the
produced for sulfuric acid Ozone Layer) is an international treaty
manufacture. designed to protect the ozone layer by
• Inhaling sulphur dioxide is associated phasing out the production of numerous
with increased respiratory symptoms substances including CFCs which are
and disease, difficulty in breathing, responsible for ozone depletion.
and premature death.
• It also weakens the functioning of 6. Consider the following statements with
certain nerves. reference to Mangroves:
• It is also produced by burning coal in 1. Mangroves exhibit Viviparity mode of
thermal power plants and diesel fuels. reproduction.

• Some industrial processes, such as 2. The mangroves of Sundarbans are the


production of paper and smelting of largest mangroves of the world.
metals also produce it. 3. Pneumatophores are the only roots
It is estimated that the emission of found in Mangrove plants.
sulfur dioxide by natural sources is 24
Which of the above is/are correct?
per cent, whereas anthropogenic
sources contribute the rest 76 per cent. A. 1 only

Statement 1 is correct: Sulfur dioxide inhibits B. 2 and 3 only


photosynthesis. C. 3 only
Statement 2 is correct: It is abundantly D. 1 and 2 only
available in the atmosphere of Venus.
Answer: D
Explanation: Odisha
Statement 1 is correct: Mangroves exhibit a • Bhaitarkanika
Viviparity mode of reproduction. i.e. seeds
• Mahanadi
germinate in the tree itself (before falling to
the ground). • Subarnarekha

Vivipary: The seed remains attached to the • Devi-Kauda


parents’ plant until it germinates. It then falls • Dhamra
from the tree and drifts on the water until it
• Mangrove Genetic Resources Centre
finds somewhere to take root. Mangroves
occur in a variety of configurations. Some • Chilka
species (e.g. Rhizophora) send arching prop Andhra Pradesh
roots down into the water. While others (e.g.
● Coringa
Avicennia) send vertical “Pneumatophores”
or air roots up from the mud. Adventitious ● East Godavari
roots that emerged from the main trunk of a ● Krishna
tree above ground level are called stilt roots.
Statement 2 is correct: The mangroves of
Sundarbans are the largest single block of 8. Consider the following statement with
reference to Mangroves for the Future
tidal halophytic mangroves of the world.
Initiative:
Statement 3 is incorrect: Mangrove plants
1. It is a non-governmental organization.
have (additional) special roots such as prop
roots, pneumatophores which help to impede 2. Its aim is to promote the study of
water flow and thereby enhance the mangroves with the purpose of
deposition of sediment in areas (where it is enhancing their conservation.
already occurring), stabilize the coastal Which of the statements given above is/are
shores, provide a breeding ground for fishes. correct?
A. 1 only
7. Which of the following Mangrove Sites is B. 2 only
not located in Odisha?
C. Both 1 and 2
A. Subarnarekha
D. Neither 1 nor 2
B. Devi-Kauda
Answer: D
C. Coringa
Explanation
D. Dhamra
Statement 1 and 2 are incorrect: Both the
Answer: C statements are about International Society for
Explanation: Mangrove Ecosystem (ISME) which is a non-
governmental organization established in
Mangrove Sites:
1990 to promote the study of mangroves with
the purpose of enhancing their conservation, Forest Products) and invasive species,
rational management and sustainable and other parameters depicting the
utilization. growth & health of the forest.
Mangroves for the Future Initiative: IUCN ● The NFI has three components: Forest
and UNDP developed a unique initiative to Inventory, TOF (Rural) Inventory, and
promote investment in coastal ecosystem TOF (Urban) Inventory.
conservation called the “Mangroves for the
Future (MFF)”. The member nations include
10. Consider the following statements with
Bangladesh, Cambodia, India, Indonesia,
reference to India State of Forest Report
Maldives, Myanmar, Pakistan, Seychelles, Sri
(ISFR):
Lanka, Thailand, and Vietnam.
1. India State of Forest Report (ISFR) is
prepared by Forest Survey of
9. Consider the following statements with India(FSI).
reference to National Forest Inventory (NFI):
2. It is an annual assessment of India’s
1. It is a major forest resource forests.
assessment activity carried out by FSI. 3. ISRO and FSI collaborates in
2. The primary objective is to assess the preparing the ISFR.
growing stock of trees Which of the statements given above is/are
3. Trees Outside Forest (Rural) correct?
Inventory is one of the components of A. 1 only
NFI.
B. 1 and 3 only
Which of the statements given above is/are C. 2 only
correct?
D. 2 and 3 only
A. 1 only
Answer: B
B. 2 and 3 only
Explanation
C. 2 only
ISFR 2021
D. 1,2 and 3
● It is the 17th biennial assessment of
Answer: D
India’s forests by the Forest Survey of
Explanation: India, an organisation under the
National Forest Inventory (NFI): Ministry of Environment, Forest and
Climate Change (MoEFCC).
● It is a major forest resource assessment
activity carried out by FSI. ● The ISFR 2021 presents the latest
status of the 'Forest cover' and 'Tree
● The primary objective is to assess the
cover' of the country, estimates of
growing stock of trees, the number of
growing stock, the extent of trees
trees, bamboo, soil carbon, the
outside forests, mangrove cover,
occurrence of NTFP (Non-Timber
bamboo resources, and assessment of 1. It is a global not-for-profit
forest carbon stock. organisation.
● In addition to the regular chapters, 2. Its major mission is the conservation
this time round, a special chapter on and restoration of marshlands.
“Forest Cover assessment in Tiger 3. The headquarters of this organisation
reserves and is in the Netherlands.
Tiger corridor areas of the country and Which of the statements given above is/are
decadal change in Forest Cover” has correct?
also been included.
A. 1 and 2 only
● Results of the two special studies
B. 2 and 3 only
namely Above Ground Biomass
C. 1 and 3 only
Estimation using Synthetic Aperture
Radar data (carried out in D. 1, 2 and 3
collaboration with ISRO) and Answer: D
Climate hot spots in forest areas Explanation
studies (carried out in collaboration
Wetlands International is a global
with BITS Pilani, Goa campus) are also
organization (NGO) that works to sustain
being presented in this report.
and restore wetlands and their resources for
● New initiatives taken up in the last people and biodiversity. Wetlands
two years by the Forest Survey of International's work ranges from research,
India (FSI) team are also being advocacy and engagement with
presented as a special chapter. governments, corporate and international
● India's progress towards achieving the policy fora and conventions
Nationally Determined Contribution Statements 1 is correct: It is an independent,
commitments are also included as part not-for-profit organization, supported by
of the chapter on Forest carbon government & NGO membership from
assessment. around the world.
Statement 1 is correct: India State of Forest
Statement 2 is correct: Its major mission is the
Report (ISFR) is prepared by Forest Survey of
conservation and restoration of marshlands.
India
Statement 3 is correct: The headquarters of
Statement 2 is incorrect: It is a biennial
this organisation is in the Netherlands.
assessment of India’s forests.
Statement 3 is correct: ISRO also collaborates
in preparing the ISFR. 12. Which of the following statement is
incorrect with reference to National Plan for
Conservation of Aquatic Ecosystems?
11. Consider the following statements with
reference to Wetlands International:
A. NPCA is a single conservation C. When the dissolved oxygen level falls
programme for both wetlands and below 3-5 ppm, many aquatic
lakes. organisms are likely to die.
B. It is a central sector scheme. D. Dissolved oxygen escapes the water
C. It was formulated by merging of the body through air-water interface and
National Lake Conservation Plan and through respiration of organisms
the National Wetlands Conservation Answer: A
Programme. Explanation
D. NPCA seeks to promote better
Dissolved Oxygen
synergy and avoid overlap of
administrative functions. • In fresh water the average
concentration of dissolved oxygen is
Answer: B
10 parts per million or 10 ppm by
Explanation weight.
Statement 1 is correct: NPCA is a single • This is 150 times lower than the
conservation programme for both wetlands concentration of oxygen in an
and lakes. equivalent volume of air.
Statement 2 is incorrect: It is a centrally • Oxygen enters the aquatic ecosystem
sponsored scheme, currently being through the air water interface and by
implemented by the Union Ministry of the photosynthetic activities of
Environment and Forests and Climate aquatic plants.
Change.
• Dissolved oxygen escapes the water
Statement 3 is correct: It was formulated in body through air-water interface and
2015 by merging of the National Lake through respiration of organisms
Conservation Plan and the National (fish, decomposers, zooplanktons,
Wetlands Conservation Programme. etc.)
Statement 4 is correct: NPCA seeks to
• The amount of dissolved oxygen
promote better synergy and avoid overlap of
retained in water is also influenced by
administrative functions.
temperature.
• Oxygen is less soluble in warm water.
13. Which of the following statement is
• Warm water also enhances
incorrect with reference to Dissolved
decomposer activity.
Oxygen?
• Therefore, increasing the temperature
A. In fresh water the average
of a water body increases the rate at
concentration of dissolved oxygen is
which oxygen is depleted from water.
more than the concentration of oxygen
in an equivalent volume of air. • When the dissolved oxygen level falls
B. Oxygen is less soluble in warm water. below 3-5 ppm, many aquatic
organisms are likely to die.
Statement 1 is incorrect: In fresh water the ● It is therefore difficult to unequivocally
average concentration of dissolved oxygen is identify the causes for bleaching
10 parts per million or 10 ppm by weight. events.
A. This is 150 times lower than the ● The following stressors have been
concentration of oxygen in an implicated in coral reef bleaching
equivalent volume of air. events.
Statement 2 is correct: Oxygen is less soluble ○ Temperature: Coral species live
in warm water. within a relatively narrow
temperature margin and
Statement 3 is correct: When the dissolved
therefore, low and high sea
oxygen level falls below 3-5 ppm, many
temperatures can induce coral
aquatic organisms are likely to die.
bleaching. Bleaching events
Statement 4 is correct: Dissolved oxygen occur during sudden
escapes the water body through air-water temperature drops
interface and through respiration of accompanying intense
organisms. upwelling episodes, seasonal
cold-air outbreaks etc.
14. Which of the following is/are the causes ○ Solar Irradiance: Bleaching
of Coral Bleaching? during the summer months,
1. Ocean acidification during seasonal temperature
and irradiance maxima often
2. Cyanide fishing occurs disproportionately in
3. Subaerial Exposure shallow-living corals and on the
exposed summits of colonies.
4. Fresh Water Dilution
○ Subaerial Exposure: Sudden
Select the correct code:
exposure of reef flat corals to the
A. 1 and 2 only atmosphere during events such
B. 3 and 4 only as extreme low tides, ENSO-
related sea level drops or
C. 4 only
tectonic uplift can potentially
D. 1,2,3 and 4 induce bleaching.
Ans: d ○ Fresh Water Dilution: Rapid
Explanation dilution of reef waters from
storm-generated precipitation
Causes of Coral Bleaching
and runoff has been
● As coral reef bleaching is a general demonstrated to cause coral
response to stress, it can be induced by reef bleaching.
a variety of factors, alone or in
○ Other causes includes the
combination.
increase in the concentration of
inorganic Nutrients, Statement C is correct: Ecophene: Population
sedimentation, oxygen which is characterized by the same genotype
starvation caused by an increase but different phenotype in a particular habitat
in zooplankton levels as a result is regarded as ecophene.
of overfishing, ocean
acidification, changes in
salinity, sea level change due to 16. Which of the following is the correct
global warming, cyanide definition of Biome?
fishing etc. A. Biome is the biological component of
earth which includes the lithosphere,
hydrosphere and atmosphere.
15. Consider the following statements:
B. Biome is a large biological community
1. Ecotone is an area that acts as a or an ecosystem where different types
boundary or a transition between two of living organisms are used to living
ecosystems. in a certain type of climate.
2. Ecotype is a plant or animal species C. The environmental gradients between
that occupy a particular habitat which two ecosystems is called Biome.
is adapted to local environmental
conditions. D. None of the above

3. Population which is characterized by Answer: B


the same genotype but different Explanation:
phenotype in a particular habitat is The terrestrial part of the biosphere isdivisible
regarded as ecophene. into enormous regions called biomes.
Which of the above is/are correct? No two biomes are alike.
A. 1 and 2 only They are characterized, by distinct
B. 2 and 3 only climate (precipitation and temperature
mainly), vegetation, animal life and general
C. 1 and 3 only
soil type.
D. 1,2 and 3
The climate determines the boundaries of a
Answer: D biome and abundance of plants and animals
Explanation found in each one of them.
Statement A is correct: An ecotone is an area Statement a is incorrect. Biosphere is
that acts as a boundary or a transition the biological component of earth which
between two ecosystems. For example, includes the lithosphere, hydrosphere
estuary, grasslands, etc. and atmosphere.
Statement B is correct: Ecotype: A plant or Statement b is correct: Biome is a
animal species that occupy a particular large biological community or an ecosystem
habitat which is adapted to local where different types of living organisms are
environmental conditions. used to living in a certain type of climate.
The climate determines the boundaries of a Statement b is incorrect. Edge Effect refers
biome and the abundance of plants and to the changes in population or
animals found in each of it. community structures that occur at the
boundary of two habitats.
Statement c is incorrect. Ecocline: There is
no sharp boundary between the two Statement c is incorrect as it explains
ecosystems, there are gradual but the Liebig's Law of the Minimum. The
continuous environmental gradients concept of limiting factors is based on Liebig's
comprised of Law of the Minimum, which states that
abiotic factors such as temperature, growth is controlled not by the total amount
slope, precipitation, water, soil, etc of resources available, but by the
and subsequently biotic communities scarcest resource
also changes. These environmental gradients
are called ecocline.
18. Animals living in polar regions are adapted to
live in subzero climates. Which of the following
17. Which of the following is the set of adaptations would not be advantageous for
correct definition of Carrying Capacity? Polar bear to survive in polar regions?
A. The carrying capacity is the A. High surface area to volume ratio
maximum population size of a B. Small extremities to reduce heat loss
particular species that a given
C. Thick layer of body fat / blubber
ecosystem can sustain.
D. Thick fur on the paws
B. Carrying capacity refers to
the changes in population or Answer: A
community structures that occur at Explanation:
the boundary of two habitats
The polar bear is a large, white bear
C. The concept of Carrying which lives in north polar regions of the earth.
capacity states that growth is The polar bear feeds mainly on fish and
controlled not by the total amount of seal. Polar bears have to adapt to live in
resources available, but by the subzero climates. They have various set
scarcest resource. of adaptations which are advantageous
D. None of the above to survive in polar regions.

Answer: A Statement A is incorrect: Low surface area


to volume ratio will be advantageous for
Explanation
polar bears to survive in polar regions.
Statement a is correct: The carrying Compared to other species of bears, polar
capacity is the maximum population size of bears have small extremities, proportionally
a particular species that a given ecosystem shorter legs and a stockier build. This reduces
can sustain. the surface area to lose heat from. This is a
common characteristic amongst animals that
live in cold environments that enables them Answer: A
to retain their heat and conversely means Explanation:
they often can't cope with warm temperatures
The organisms try to maintain the constancy
as they easily overheat in warmer
of their internal environment (a process
temperatures.
called homeostasis) despite varying
Statement B is correct: Small extremities external environmental conditions that tend
to reduce heat loss (anatomical) - Polar to upset homeostasis.
bears have small ears covered in thick fur, a
Homeostasis can be defined as the stable state
very short tail and a relatively short and
of an organism and of its internal
squat muzzle, all of which help to retain heat.
environment; as the maintenance or
Statement C is correct: Thick layer of body regulation of the stable condition, or its
fat / blubber [upto 10cm (4 inches) thick] is equilibrium; or simply as the balance of
used both for insulation and also for food bodily functions.
storage to help them to survive when food
Statement a is correct: Homeostasis is
supply may be intermittent especially in the
the state of steady internal chemical and
summer months.
physical conditions maintained by living
Statement D is correct: Thick fur on the systems.
paws are to insulate them from snow and ice
Statement b is incorrect. It is the definition of
and also provide for grip on slippery
Ecological succession. The process by
surfaces. The paws are large in size to help
which communities of plant and animal
spread the bears weight over ice and snow.
species in an area are replaced or changed
into another over a period of time is known as
19. Which of the following is the ecological succession.
correct definition of Homeostasis? Statement c s incorrect as it is the definition of
A. Homeostasis is the state of Haemostasis. It is the body's natural reaction
steady internal chemical and to an injury that stops bleeding and repairs
physical conditions maintained by the damage
living systems.
B. Homeostasis is the process by 20. Which one of the following is the
which communities of plant and correct sequence of ecosystems in the order of
animal species in an area are replaced decreasing productivity? [ UPSC 2013]
or changed into another over a period
A. Oceans, lakes, grasslands, mangroves
of time
B. Mangroves, oceans, grasslands, lakes
C. Homeostasis is the body's
natural reaction to an injury that C. Mangroves, grasslands, lakes, oceans
stops bleeding and repairs the D. Oceans, mangroves, lakes, grasslands
damage
Answer: C
D. None of the above
Explanation:
Ecosystems in the order of awareness of their importance has
decreasing productivity: grown.
Mangroves > Grasslands > Lakes > Oceans Additional Information:
What is a Carbon Sink?
21. Consider the following statements with • A carbon sink is any natural or man-
reference to Carbon Sink: made reservoir that accumulates and
stores some carbon-containing
1. A carbon sink is any natural reservoir
chemical compound for an indefinite
only
period of time, lowering CO2
2. Soil is a critical carbon storage concentrations in the atmosphere.
medium.
• The two most important carbon sinks
3. Kyoto Protocol is associated with CO2 on a global scale are vegetation and the
sinks ocean.
Which of the above is/are correct? • Since the passage of the Kyoto
A. 1 only Protocol, which encourages the use of
CO2 sinks as a form of carbon offset,
B. 2 and 3 only
public awareness of their importance
C. 2 only has grown.
D. 1, 2 and 3 • Soil is a critical carbon storage
Answer: B medium. Due to intensive farming,
much of the organic carbon retained in
Explanation:
agricultural areas has been depleted.
• Statement 1 is incorrect: A carbon sink
• "Blue carbon" refers to carbon fixed by
is any natural or man-made reservoir
ocean ecosystems. The majority of
that accumulates and stores some
ocean plant life is made up of
carbon-containing chemical
mangroves, salt marshes, and
compound for an indefinite period of
seagrasses, which store large amounts
time, lowering CO2 concentrations in
of carbon.
the atmosphere.
• Numerous efforts are being made to
• Statement 2 is correct: Soil is a critical
improve natural sequestration in soils
carbon storage medium. Due to
and oceans.
intensive farming, much of the
organic carbon retained in • Furthermore, a variety of artificial
agricultural areas has been depleted. sequestration initiatives, such as new
building materials, carbon capture and
• Statement 3 is correct: Since the
storage, and geological sequestration,
passage of the Kyoto Protocol, which
are underway.
encourages the use of CO2 sinks as a
form of carbon offset, public
22. Which of the following statements with • Established by World Meteorological
reference to IPCC’s Assessment Reports Organisation (WMO) and United
(ARs) is incorrect? Nations Environment Programme
A. The first Assessment Report formed (UNEP) in 1988
the basis for the negotiation of the UN • Membership: Open for all the
Framework Convention on Climate members of the WMO and UN.
Change (UNFCCC)
• Function of IPCC:
B. The second Assessment Report was
o It provides objective scientific
the scientific underpinning for the
information in order to
Kyoto Protocol
understand human- induced
C. The third Assessment Report won the climate change.
Nobel Peace Prize for IPCC
▪ It also covers natural, political &
D. The fourth Assessment Report was economic impacts of these
the scientific input for the 2009 anthropogenic climate changes and
Copenhagen climate meeting possible response options.
Answer: C ▪ Its main activity is to prepare
Explanation: Assessment Reports, special
reports, and methodology reports
• Statement 1 is correct: The first
assessing the state of knowledge
Assessment Report formed the basis
of climate change.
for the negotiation of the UN
Framework Convention on Climate ▪ However, the IPCC does not itself
Change (UNFCCC) engage in scientific research. Instead, it
asks scientists from around the world
• Statement 2 is correct: The second
to go through all the relevant scientific
Assessment Report was the scientific
literature related to climate change and
underpinning for the Kyoto Protocol
draw up the logical conclusions.
• Statement 3 is incorrect: The fourth
▪ It does not monitor climate or
Assessment Report won the Nobel related phenomena itself.
Peace Prize for IPCC
Assessment Reports
• Statement 4 is correct: The fourth
• The IPCC’s Assessment Reports (ARs),
Assessment Report was the scientific
which are produced every few years,
input for the 2009 Copenhagen
are the most comprehensive and
climate meeting
widely accepted scientific evaluations
Additional Information: of the state of the Earth’s climate.
Intergovernmental Panel on Climate • They form the basis for government
Change (IPCC)
policies to tackle climate change, and
• An intergovernmental body of the provide the scientific foundation for
United Nations (UN).
the international climate change degrees Celsius by 2100. Sea
negotiations. levels were likely to rise by 65
cm by 2100.
o Six Assessment Reports have been
published so far ▪ This report formed the basis for the
negotiation of the UN Framework
▪ The sixth report (AR6) is coming
Convention on Climate Change
in three parts — the first in August
(UNFCCC) in 1992, known as the Rio
2021, the second in February 2022,
Summit.
and the third recently .
• The second Assessment Report (1995):
▪ The first part of AR6 flagged more
intense and frequent heat-waves, o It revised the projected rise in
increased incidents of extreme global temperatures to 3
rainfall, a dangerous rise in sea- degrees Celsius above pre-
levels, prolonged droughts, and industrial levels by 2100, and
melting glaciers — and said that sea-level rise to 50 cm, in light of
1.5 degrees Celsius warming was more evidence. Global rise in
much closer than was thought temperature by 0.3 to 0.6 degree
earlier, and also inevitable. Celsius since the late 19th
century was “unlikely to be
▪ The second part warned that multiple
entirely natural in origin”, it
climate change-induced disasters were
said.
likely in the next two decades even if
strong action was taken to reduce the o AR2 was the scientific
emissions of greenhouse gases. underpinning for the Kyoto
Protocol of 1997.
• What previous reports have said
• The third Assessment Report (2001)
▪ First Assessment Report
o It revised the projected rise in
▪ The first Assessment Report
global temperatures to 1.4 to 5.8
(1990) noted that emissions
degrees Celsius by 2100
resulting from human activities
compared to 1990.
are substantially increasing the
atmospheric concentrations of o The projected rate of warming
greenhouse gases. was unprecedented in the last
10,000 years, it said.
▪ Global temperatures have risen
by 0.3 to 0.6 degree Celsius in o The report predicted increased
the last 100 years. rainfall on average, and that by
2100, sea levels were likely to
▪ In the business-as-usual
rise by as much as 80 cm from
scenario, temperatures were
1990 levels.
likely to increase by 2 degrees
Celsius compared to pre- o Glaciers would retreat during
industrial levels by 2025, and 4 the 21st century, and the
frequency, intensity, and “large fraction of species” faced
duration of extreme weather extinction, and food security
events would increase, it said. would be undermined.
The report presented new and o AR5 formed the scientific basis
stronger evidence to show for negotiations of the Paris
global warming was mostly Agreement in 2015.
attributable to human activities.
• The fourth Assessment Report (2007):
23. REDD+ activities include which of the
o It said greenhouse gas following?
emissions increased by 70 per
1. Reducing emissions from
cent between 1970 and 2004,
deforestation
and atmospheric concentrations
of CO2 in 2005 (379 ppm) were 2. Reducing emissions from forest
the most in 650,000 years. degradation
o In the worst-case scenario, 3. Conservation of forest carbon stocks
global temperatures could rise 4. Sustainable management of forests
4.5 degrees Celsius by 2100
from pre-industrial levels, and 5. Enhancement of forest carbon stocks
sea levels could be 60 cm higher Select the correct code:
than 1990 levels. A. 1 and 2 only
o The report won the 2007 Nobel B. 4 and 5 only
Peace Prize for IPCC. It was the
scientific input for the 2009 C. 2, 3, and 4 only
Copenhagen climate meeting. D. 1, 2, 3, 4 and 5
• The fifth Assessment Report (2014) Answer: D
o It said more than half the Explanation:
temperature rise since 1950 was The differences between REDD and REDD+
attributable to human activities, are tabled below:
and that the atmospheric
concentrations of carbon REDD REDD+
dioxide, methane, and nitrous
oxide were “unprecedented” in
the last 800,000 years. REDD is the REDD+, with the
abbreviation for “plus” referring to
o The rise in global temperatures
“reducing “the role of
by 2100 could be as high as 4.8
emissions from conservation,
degrees Celsius from pre-
deforestation and sustainable
industrial times, and more
forest degradation” management of
frequent and longer heat waves
forests and
were “virtually certain”. A
Answer: D
enhancement of
forest carbon stocks Explanation:
in developing • Statement 1 is correct: Get its non-
countries”. fossil energy capacity to 500 gigawatts
by 2030.
REDD is mostly REDD+ activities • Statement 2 is correct: Meet 50 per
focused towards include the cent of its energy requirements till
avoiding carbon following: 2030 with renewable energy.
emissions from: • Reducing • Statement 3 is correct: Reduce its
• deforestatio emissions projected carbon emission by one
n and from billion tonnes by 2030.
deforestation
• forest • Statement 4 is incorrect: Achieve net
degradation • Reducing zero by 2070.
emissions
Additional Information:
from forest
degradation What is Panchamrita?

• Conservation • ‘Panchamrita’ is a traditional method


of forest of mixing five natural foods — milk,
carbon stocks ghee, curd, honey and jaggery. These
are used in Hindu and Jain worship
• Sustainable
rituals. It is also used as a technique in
management
Ayurveda.
of forests
• The PM euphemistically termed his
• Enhancemen
scheme as ‘Panchamrita’ meaning the
t of forest
‘five ambrosia’.
carbon stocks
• Under Panchamrita’, India will:
o Get its non-fossil energy capacity
24. Which of the following is not India's to 500 gigawatts by 2030.
target under Panchamrita? o Meet 50 per cent of its energy
A. Get its non-fossil energy capacity to requirements till 2030 with
500 gigawatts by 2030 renewable energy.

B. Meet 50 per cent of its energy o Reduce its projected carbon


requirements till 2030 with renewable emission by one billion tonnes by
energy 2030.

C. Reduce its projected carbon emission o Reduce the carbon intensity of its
by one billion tonnes by 2030 economy by 45 per cent by 2030.

D. Achieve net zero by 2050 o Achieve net zero by 2070.


Net Zero Emissions
25. Which of the following statements with • It is also referred to as carbon-
reference to carbon-neutrality is incorrect? neutrality.
A. It is also referred to as Net Zero • It does not mean that a country would
Emissions bring down its emissions to zero.
B. It means that a country would bring • Rather, net-zero is a state in which a
down its emissions to zero. country’s emissions are compensated
C. India has not yet committed to a net by absorption and removal of
zero timeline. greenhouse gases from the
atmosphere.
D. Bhutan is often described as carbon-
negative • Absorption of the emissions can be
increased by creating more carbon
Answer: B
sinks such as forests, while removal of
Explanation: gases from the atmosphere requires
• Statement 1 is correct: It is also futuristic technologies such as carbon
referred to as Net Zero Emissions capture and storage.
• This way, it is even possible for a
• Statement 2 is incorrect: It does not
mean that a country would bring country to have negative emissions, if
down its emissions to zero. Rather, the absorption and removal exceed the
net-zero is a state in which a country’s actual emissions.
emissions are compensated by • A good example is Bhutan which is
absorption and removal of often described as carbon-negative
greenhouse gases from the because it absorbs more than it emits.
atmosphere.
• India is currently the World’s third
• Statement 3 is correct: India has not biggest emitter of greenhouse gases,
yet committed to a net zero timeline. after the US and China.
Though India is not against 'net zero' • India has not yet committed to a net
goal per se, the country will not zero timeline.
commit to the 2050 timeline. It will
strongly underline its demand for the • Several other countries, including the
developed world sticking to the UK and France, have already enacted
principle of common but laws promising to achieve a net-zero
differentiated responsibility and the emission scenario by the middle of the
need to deliver on climate change century.
mitigation finance. • The European Union is working on a
similar Europe-wide law, while many
• Statement 4 is correct: Bhutan is often
other countries including Canada,
described as carbon-negative.
South Korea, Japan and Germany have
Additional Information:
expressed their intention to commit Additional Information:
themselves to a net-zero future. Kyoto Protocol
• Even China has promised to go net- • 192 countries are parties to the Kyoto
zero by 2060. Protocol.
• 84 countries are signatories of the
26. Which of the following statements is Kyoto Protocol.
incorrect with reference to the Kyoto • Canada, Andorra, The United States of
Protocol?
America, and South Sudan are not
A. Canada, Andorra, The United States parties to the protocol.
of America, and South Sudan are not
• It is legally binding and only members
parties to the protocol.
of the UNFCCC can become parties to
B. It is legally binding and only members the Kyoto Protocol.
of the UNFCCC can become parties to
• It was adopted at the 3rd session of
the Kyoto Protocol.
UNFCCC
C. To meet the targets of the Kyoto
• To meet the targets of the Kyoto
Protocol, member countries can
Protocol, member countries cannot
include international shipping and
include international shipping and
international aviation
international aviation
D. Countries can use Land Use (LU),
• Countries can use Land Use (LU),
land-use change (LUC), and Forestry
land-use change (LUC), and Forestry
to meet their Kyoto targets.
to meet their Kyoto targets.
Answer: C
• China signed the protocol in 1998.
Explanation:
• The protocol did not include ozone-
Statement 1 is correct: Canada, Andorra, The depleting substances which were later
United States of America, and South Sudan covered by the Montreal protocol.
are not parties to the protocol.
Statement 2 is correct: It is legally binding
27. Consider the following statements with
and only members of the UNFCCC can
reference to Carbon budgets:
become parties to the Kyoto Protocol.
1. A carbon budget is a cumulative
Statement 3 is incorrect: To meet the targets amount of carbon dioxide (CO2)
of the Kyoto Protocol, member countries emissions permitted over a period of
cannot include international shipping and time to keep within a certain
international aviation temperature threshold.
Statement 4 is correct: Countries can use 2. Carbon budgets are constructed on the
Land Use (LU), land-use change (LUC), and premise that there is a near-linear
Forestry to meet their Kyoto targets. relationship between rising global
temperatures and the level of • But with carbon building up quickly,
cumulative atmospheric CO2. countries are having to redraw their
Which of the above is/are correct? budgets more often than hoped.

A. 1 only • A carbon budget is the cumulative


amount of carbon dioxide (CO2)
B. 2 only emissions permitted over a period of
C. Both 1 and 2 time to keep within a certain
temperature threshold. Figure 1 shows
D. Neither 1 nor 2
a range of carbon budgets as published
Answer: C by different institutions in the energy
Explanation: and climate change sector that it is
Statement 1 is correct: projected will keep average increases
in global temperature to within 2°C. As
• A carbon budget is a cumulative it is, these 2°C budgets are not
amount of carbon dioxide (CO2) immediately comparable, however.
emissions permitted over a period of
time to keep within a certain
temperature threshold.
• It is the maximum amount of carbon
dioxide (CO2) that can be emitted
while still having a chance to limit
warming to 1.5°C or 2°C.
Statement 2 is correct: Carbon budgets are
constructed on the premise that there is a
near-linear relationship between rising global
temperatures and the level of cumulative
atmospheric CO2. 28. Consider the following pairs of Climate
groupings:
Additional info:
1. Umbrella Group: This alliance of non-
How do carbon budgets work?
EU developed countries includes
• Carbon budgets measure how much Australia, Japan, Russia, and the
CO2 is produced by industry, homes United States.
and all other parts of the economy to
2. Africa Group: Africa’s UN members.
calculate how much emissions must be
cut in the future. 3. Climate Vulnerable Forum:
Representing 58 countries most at risk
• The aim is to reach net-zero emissions
from climate impacts.
– striking an equal balance between the
carbon released into the atmosphere Which of the above is/are correctly matched?
and that removed from it. A. 1 only
B. 2 and 3 only
C. 3 only 29. The Paris Agreement talks about 20/20/20
targets. Which of them is/are a part of it?
D. 1,2 and 3
1. Carbon Dioxide emissions reductions
Answer: D
by 20%,
Explanation:
2. Work on increasing the renewable
• Umbrella Group: This alliance of non- energy market share by 20%
EU developed countries includes
3. Target to increase energy efficiency by
Australia, Japan, Russia, and the
20%
United States.The Umbrella Group is a
loose coalition of non-EU developed Select the correct code:
countries which formed following the A. 1 and 2 only
adoption of the Kyoto Protocol.
Although there is no formal list, the B. 2 and 3 only
Group is usually made up of Australia, C. 1 and 3 only
Canada, Japan, New Zealand,
D. 1, 2 and 3
Kazakhstan, Norway, the Russian
Federation, Ukraine and the US. Answer: D

• Africa Group: Africa’s UN members. Explanation:


The Group of African States, or African The goal of the Paris Agreement
Group, is one of the five United 1. To curtail the rise of global
Nations regional groups and is temperature this century below 2-
composed of 54 Member States from degree Celsius, above pre-industrial
the African continent. The African levels; and also pursue efforts to limit
Group is the largest regional group, the increase to 1.5 degrees Celsius.
and composes 28% of all United
Nations members. 2. Develop mechanisms to help and
support countries that are very
• Climate Vulnerable Forum: vulnerable to the adverse impacts of
Representing 58 countries most at risk
climate change. An example would be
from climate impacts. The Climate
countries like the Maldives facing
Vulnerable Forum is a global
threat due to sea-level rise.
partnership of countries that are
disproportionately affected by the 3. Confirms the obligation that
consequences of climate change. The developed countries have towards
forum addresses the negative effects of developing countries, by providing
climate change as a result of them financial and technological
heightened socioeconomic and support.
environmental vulnerabilities The agreement talks about 20/20/20 targets,
i.e.
1. Carbon Dioxide emissions reductions
by 20%,
2. Work on increasing the renewable What is a Like-Minded Developing
energy market share by 20% Countries (LMDC) group?
3. Target to increase energy efficiency by • LMDC comprises around 25
20% developing countries from Asia and
other regions.
• It organizes themselves as a block of
30. Consider the following statements with
reference to Like-Minded Developing negotiators in international
Countries(LMDC) group: organizations such as the United
Nations.
1. LMDC comprises around 25
developing countries from Asia and • They represent more than 50% of the
other regions. world’s population.

2. It organizes themselves as a block of • Member countries: Algeria,


negotiators in international Bangladesh, Belarus, Bhutan, China,
organizations such as the United Cuba, Egypt, India, Indonesia, Iran,
Nations. Malaysia, Myanmar, Nepal, Pakistan,
the Philippines, Sri Lanka, Sudan,
3. They represent more than 50% of the
Syria, Vietnam, and Zimbabwe.
world’s population.
Which of the above is/are correct?
31. Consider the following statements with
A. 1 only reference to Powering Past Coal Alliance:
B. 2 and 3 only 1. The Powering Past Coal Alliance or
C. 2 only PPCA is a group that aims to reduce
coal power.
D. 1, 2 and 3
2. The main objective of the alliance is to
Answer: D
phase out coal fired power plants.
Explanation:
3. It was initially launched by Canada
Statement 1 is correct: LMDC comprises and the United Kingdom.
around 25 developing countries from Asia
and other regions. 4. The Government of Canada provides
financial support to the project.
Statement 2 is correct: It organizes
Which of the statements given above is/are
themselves as a block of negotiators in
correct?
international organizations such as the
United Nations. A. 1 and 2 only

Statement 3 is correct: They represent more B. 3 and 4 only


than 50% of the world’s population. C. 2 only
Additional info: D. 1, 2, 3 and 4
Answer: D
Explanation: A. 1 only
Statement 1 is correct: The Powering Past B. 2 only
Coal Alliance or PPCA is a group that aims to
C. Both 1 and 2
reduce coal power.
D. Neither 1 nor 2
Statement 2 is correct: The main objective of
the alliance is to phase out coal fired power Answer: C
plants. Explanation:
Statement 3 is correct: It was initially Statement 1 is correct: SDM must contribute
launched by Canada and the United to overall emission reductions/net mitigation
Kingdom.
Statement 2 is correct: CDM is based on
Statement 4 is correct: The Government of Kyoto Protocol
Canada provides financial support to the
Additional info:
project.
Additional info:
• The Powering Past Coal Alliance or
PPCA is a group that aims to reduce
coal power. The main objective of the
alliance is to phase out coal fired power
plants.
• It was initially launched by Canada
and the United Kingdom.
• It was launched during the 2017
UNFCCC (United Nations Convention
on Climate Change) COP23 held in 33. Which among the following does not
Bonn. The Government of Canada manage the Global Environment Facility ?
provides financial support to the
A. United Nations Development
project.
Programme (UNDP)
B. World Meteorological Organization
32. Consider the following statements with (WMO)
reference to SDM and CDM building
C. World Bank
blocks:
D. United Nations Environment
1. Sustainable Development Mechanism
Programme (UNEP)
(SDM) must contribute to overall
emission reductions/net mitigation Answer: B

2. Clean Development Mechanism Explanation:


(CDM) is based on Kyoto Protocol Global Environment Facility (GEF): Since its
Which of the above is/are correct? inception, this multilateral financial
mechanism has advanced $21.1 billion in formed a Permanent Constituency in
grants and mobilized an additional $114 the Executive Council of the GEF.
billion in financing for more than 5,000 4. India receives funding from the GEF
projects in 170 countries. for work in three major areas namely,
• GEF was established during the Rio biodiversity, climate change, and land
Earth Summit of 1992. degradation.
• It is based in Washington, D.C., Which of the statements given above is/are
United States. correct?
• The GEF is jointly managed by the A. 1 and 2 only
United Nations Development B. 3 and 4 only
Programme (UNDP), the World Bank,
C. 1 and 3 only
and the United Nations Environment
Programme (UNEP). D. 2 and 4 only
• 183 nations are united under GEF in Answer: B
partnership with civil society Explanation:
organizations (CSOs), international
Statement 1 is incorrect: India is both a donor
institutions, private sector, etc. to
and a recipient of funding from the GEF.
address the environmental issues
across the globe. Statement 2 is incorrect: The political focal
point for GEF in India is the Finance Ministry
• The financial mechanism was
while the operational focal point is the
established to help tackle our planet’s
Environment Ministry.
most pressing environmental
problems. It provides funds to the Statement 3 is correct: India, Bhutan,
developing countries and transition Maldives, Sri Lanka, Nepal, and Bangladesh
economies for projects related to have together formed a Permanent
climate change, biodiversity, the Constituency in the Executive Council of the
ozone layer, etc. GEF.
Statement 4 is correct: India receives funding
from the GEF for work in three major areas
34. Consider the following statements with
reference to GEF and India: namely, biodiversity, climate change, and
land degradation.
1. India is only a donor of funding to the
GEF.
2. The political focal point for GEF in 35. According to ‘Emissions Gap Report
2022’ which of the following countries is not
India is the Environment Ministry
among the top 7 emitters?
while the operational focal point is the
Finance Ministry. A. Indonesia

3. India, Bhutan, Maldives, Sri Lanka, B. Brazil


Nepal, and Bangladesh have together C. Australia
D. Russian Federation Statement 2 is correct: But the UK and
Answer: C Indonesia are going to decrease their oil and
gas output.
Explanation:
Statement 3 is correct: Denmark also decides
• The top 7 emitters (China, the EU27,
to cancel all future licensing rounds for oil
India, Indonesia, Brazil, the Russian
and gas, and completely phase out oil
Federation and the United States of
production by 2050.
America) plus international transport
accounted for 55% of global GHG Additional info:
emissions in 2020. • The Production Gap Report — first
launched in 2019 — tracks the
discrepancy between governments’
36. Consider the following statements with planned fossil fuel production and
reference to findings of the Production Gap
global production levels consistent
Report:
with limiting warming to 1.5°C or 2°C.
1. There is an increase in oil and gas
• The report represents a collaboration
production in the United States,
of several researchers and academic
Canada, Australia, Saudi Arabia and
institutions, including input from
China.
more than 40 experts. UNEP staff
2. But the UK and Indonesia are going to provided guidance and insights from
decrease their oil and gas output. their experience
3. Denmark also decides to cancel all • This year’s report presents the first
future licensing rounds for oil and gas, comprehensive update of the
and completely phase out oil production gap analysis since our 2019
production by 2050. assessment.
Which of the above is/are correct? • The report also tracks how
A. 1 and 2 only governments worldwide are
supporting fossil fuel production
B. 2 and 3 only
through their policies, investments,
C. 1 and 3 only and other measures, as well as how
D. 1, 2 and 3 some are beginning to discuss and
enact policies towards a managed and
Answer: D
equitable transition away from fossil
Explanation: fuel production.
Statement 1 is correct: There is an increase in • This year’s report features individual
oil and gas production in the United States, country profiles for 15 major fossil
Canada, Australia, Saudi Arabia and China. fuel-producing countries and a special
chapter on the role of transparency in
helping to address the production gap.
3. Ocean acidification reduces the
37. Greenhouse Gas Bulletin Report shows amount of carbonate which is a key
the average surface mole fractions for which building block in seawater.
of the following? Which of the statements given above is/are
1. Carbon dioxide (CO2) correct?

2. Nitrous oxide (N2O) A. 1 only

3. Carbon monoxide (CO) B. 2 and 3 only

Select the correct code: C. 2 only

A. 1 and 2 only D. 1 and 3 only

B. 2 only Answer: D

C. 1 and 3 only Explanation:

D. 1, 2 and 3 • Statement 1 is correct: Ocean


acidification has the potential to
Answer: A impact food security.
Explanation:
• Statement 2 is incorrect: As the ocean
• Greenhouse Gas Bulletin Report shows acidifies, its ability to absorb CO2
the average surface mole fractions for decreases.
carbon dioxide (CO2), methane (CH4)
• Statement 3 is correct: Ocean
and nitrous oxide (N2O) and
acidification reduces the amount of
compares them with the mole fractions
carbonate which is a key building
during the previous year and with the
block in seawater.
preindustrial levels.
Additional Information:
• Greenhouse Gas Bulletin 2022 also
provides change in radiative forcing by Carbon Storage and Climate Regulation
long-lived GHGs (LLGHGs) and the • As the ocean acidifies, its ability to
contribution of individual gases to this absorb CO2 decreases.
increase.
• More acidic oceans are less effective at
mitigating climate change.
38. Consider the following statements with Food
reference to Ocean Acidification:
• Ocean acidification has the potential to
1. Ocean acidification has the potential to impact food security.
impact food security.
• The global annual costs of mollusk loss
2. As the ocean acidifies, its ability to due to ocean acidification could exceed
absorb CO2 increases. $100 by 2100.
• When fish die, humans who rely on some types of plankton. These animals'
them for food and a living suffer from shells and skeletons may become less
socioeconomic problems. dense or strong.
• This process also contributes to acidic • This may make coral reefs more
waters, which harm agricultural vulnerable to storm damage and slow
production even more. their recovery rate.
• Soil acidity rises as a result of acidic • In response to ocean acidification,
water. This makes the cultivation and marine organisms may experience
production of certain crops impossible. changes in growth, development,
Effect of Ocean Acidification on Marine abundance, and survival.
Organisms and Ecosystem • Most species appear to be more
• Ocean acidification reduces the vulnerable in their early stages of
amount of carbonate, a key building development. Juvenile fish, for
block in seawater. example, may have difficulty finding
suitable habitat to live in.
• This makes it more difficult for marine
organisms, such as coral and some • Regardless of whether the responses
plankton, to form shells and skeletons, within and between marine groups are
and existing shells may begin to positive or negative, research indicates
dissolve. that ocean acidification will be a driver
of significant changes in ocean
• The pH of seawater is highly variable,
ecosystems this century.
and a single organism can cope with
fluctuations of different pH levels over • These changes may be exacerbated by
its lifetime. the interaction with other emerging
climate-related hazards, such as the
• The issue with ocean acidification is
decrease in ocean oxygen levels,
the long-term nature of the change, as
known as ocean deoxygenation, which
the risk comes from lifetime exposure
is already affecting marine life in some
to lower pH levels. The rate of
areas.
acidification will affect the ability of
calcifying organisms to adapt.
• The effects of ocean acidification are 39. Which of the following is/are Natural
not the same for all species. Higher Causes of Global Warming?
CO2 concentrations in the ocean may 1. Volcanoes
benefit some algae and seagrass by 2. Water Vapour
increasing photosynthetic and growth
rates. 3. Melting Permafrost

• A more acidic environment, on the 4. Forest Fires


other hand, will harm other marine Select the correct code:
species such as mollusks, corals, and
A. 1 and 2 only
B. 3 only reef is the Great Barrier Reef
C. 4 only C. Atolls are far more common in the
Pacific than any other ocean.
D. 1,2,3 and 4
Answer: D D. None of the above

Explanation: Answer: D

Natural Causes of Global Warming Explanation:

• Volcanoes: Volcanoes are one of the • Statement 1 is correct: Fringing Reef is


most significant natural contributors to a coral platform attached to a
global warming. The ash and smoke continental coast or an island
emitted by volcanic eruptions enter the • Statement 2 is correct: The most
atmosphere and affect the climate. famous example of barrier reef is the
• Water Vapour: Water vapour is a type Great Barrier Reef
of greenhouse gas. As the earth's • Statement 3 is correct: Atolls are far
temperature rises, more water more common in the Pacific than any
evaporates from bodies of water and other ocean.
enters the atmosphere, contributing to
Additional Information:
global warming.
Fringing Reef
• Melting Permafrost: Permafrost is
frozen soil that has been trapped in it • It is by far the most common of the
for several years and is found beneath three major types of coral reefs.
the Earth's surface. It can be found in • It is a coral platform attached to a
glaciers. As permafrost melts, gases are continental coast or an island,
released back into the atmosphere, sometimes separated by a narrow,
raising Earth's temperature. shallow lagoon or channel.
• Forest Fires: Forest fires emit a large • A fringing reef runs as a narrow belt,
amount of carbon-containing smoke. 0.5 km to 2.5 km wide.
These gases are released into the
• This type of reef grows from the deep
atmosphere and raise the temperature
sea bottom with the seaward side
of the Earth, resulting in global
sloping steeply into the deep sea.
warming.
• Coral polyps do not extend outwards
because of sudden and large increases
40. Which of the following statements is in depth.
incorrect with respect to Coral reefs?
• The surface of a fringing reef is rough,
A. Fringing Reef is a coral platform as it is covered with coral remains
attached to a continental coast or an forming a boulder zone or reef flat.
island
Barrier Reef
B. The most famous example of barrier
• This is the largest of the three reefs, which may reach a level suitable for
runs for hundreds of kilometres and is coral growth.
several kilometres wide. • Atolls are far more common in the
• It extends as a broken, irregular ring Pacific than any other ocean. The Fiji
around the coast, or an island, running atoll and the Funafuti atoll in the Ellice
almost parallel to it. Island are well-known examples of
atolls. A large number of atolls also
• A barrier reef is characterized by the
occur in the Lakshadweep islands.
distant location of the reef from the
coast with a broader and deeper • In the South Pacific, most atolls occur
lagoon, which is sometimes joined in mid-ocean. Examples of this reef
with the seawater through one or more type are common in French Polynesia,
channels cutting across the barrier reef. the Caroline and Marshall Islands,
Micronesia, and the Cook Islands.
• A barrier reef is very thick, going even
below 180 metres from the surface with • The Indian Ocean also contains
the seaward side sloping steeply into numerous atoll formations. Examples
the deep sea. The surface of a barrier are found in the Maldives and Chagos
reef is covered with coral debris, island groups, the Seychelles, and in
boulders, and sand. the Cocos Island group.
• The most famous example of this type
of reef is the Great Barrier Reef off the 41. Consider the following statements:
coast of northeastern Australia, which
is 1900 km long and 160 km wide. 1. Dry deposition results in the return of
around half of the atmosphere's acidity
Atolls to the planet.
• It is a ring-like reef, which, partly or 2. Acid rain containing ions of sulfate,
completely, encloses a lagoon. The nitrate, ammonium and hydrogen falls
lagoon may have a level surface, but as wet deposition.
the seaward side of the reef slopes
steeply into the deep sea. 3. Rainwater is naturally slightly acidic

• The lagoon has a depth of 80-150 Which of the statements given above is/are
metres and may be joined with correct?
seawater through a number of A. 1 only
channels cutting across the reef. B. 2 and 3 only
• Atolls are located at great distances C. 3 only
from deep-sea platforms, where the
submarine features may help in D. 1, 2 and 3
formation of atolls, such as a Answer: D
submerged island or a volcanic cone Explanation:
• Statement 1 is correct: Dry deposition higher GWP absorb more energy per
results in the return of around half of pound than gases with a lower GWP,
the atmosphere's acidity to the planet. contributing more to global warming.

• Statement 2 is correct: Acid rain • Statement 2 is correct: CO2 has a GWP


containing ions of sulfate, nitrate, of 1. Other gases are affected by the gas
ammonium and hydrogen falls as wet and the time frame.
deposition. Additional Information:
• Statement 3 is correct: CO and CO2 What is Global Warming Potential?
react with rainwater to form weak
• The global warming potential (GWP) is
carbonic acid. Hence, rainwater is
the heat absorbed by any greenhouse
naturally slightly acidic. But this is not
gas in the atmosphere expressed as a
enough to call it acid rain (acid rain
multiple of the heat absorbed by the
must have a pH of less than 5.6). Even
same mass of carbon dioxide (CO2).
increased concentration of CO and
CO2 is not enough to cause rainwater • The global warming potential of each
of pH less than 5.6. Only NIOS (10.3.2 gas describes its impact on global
Gaseous pollutants > Table 10.3 – Page warming.
167) mentions N2O (nitrous oxide). • In terms of climate impact, the two
most important characteristics of a
GHG are:
42. Consider the following statements with
reference to Global Warming Potential o how well the gas absorbs
(GWP): energy (preventing it from
1. Gasses with a higher GWP absorb immediately escaping to space);
more energy per pound than gasses o how long the gas remains in the
with a lower GWP, contributing more atmosphere.
to global warming.
• The Global Warming Potential (GWP)
2. CO2 has a GWP of 1 of a gas is a measure of how much
Which of the statements given above is/are energy a gas absorbs over a specific
correct? time period (usually 100 years) when
compared to carbon dioxide.
A. 1 only
• Gases with a higher GWP absorb more
B. 2 only
energy per pound than gases with a
C. Both 1 and 2 lower GWP, contributing more to
D. Neither 1 nor 2 global warming.

Answer: C • CO2 has a GWP of 1. Other gases are


affected by the gas and the time frame.
Explanation:
• GWP is used to calculate carbon
• Statement 1 is correct: Gases with a
dioxide equivalent. The mass of CO2
would warm the earth as much as the 44. Consider the following statements with
mass of any other gas. respect to Carbon Monoxide:
• As a result, it provides a common scale 1. It is a colorless, odorless, tasteless, and
for assessing the climate effects of highly toxic gas.
various gases. It is calculated as GWP 2. It is short-lived in the atmosphere.
times the mass of the other gas.
3. It is produced from the exhaust of
internal combustion engines and from
43. Consider the following statements: incomplete combustion of various
1. Primary pollutants are directly emitted other fuels.
into the air from the Earth’s surface. Which of the statements given above is/are
correct?
2. Secondary pollutants react chemically
in the atmosphere to form other A. 1 and 2
compounds. B. 2 and 3
Which of the above statements given is/are C. 1 and 3
correct?
D. All of the above.
A. 1 only
Answer: - D
B. 2 only
Explanation:
C. Both 1 and 2
All of the above are correct.
D. None of the above.
Carbon monoxide (CO)
Answer: - D
• Carbon monoxide (CO) is a colourless,
Explanation: odourless, tasteless and highly toxic
• The examples of both the types of gas that is slightly less dense than air.
pollutants are interchanged. It is short-lived (stay only a few
months) in the atmosphere.
• Primary pollutants: These are
persistent in the form in which they are • Carbon monoxide is produced from
added to the environment, e.g. DDT, the exhaust of internal combustion
plastic, CO, CO2, oxides of nitrogen engines and incomplete combustion of
and sulphur, etc. various other fuels. Iron smelting also
produces carbon monoxide as a by-
• Secondary Pollutants: These are
product.
formed by interaction among the
primary pollutants. For example, • It forms when there is not enough
peroxyacetyl nitrate (PAN) is formed oxygen to produce carbon dioxide
by the interaction of nitrogen oxides (CO2).
and hydrocarbons. • In the presence of oxygen, carbon
monoxide burns with a blue flame,
producing carbon dioxide.
• Worldwide, the largest source of Select the correct code from the options given
carbon monoxide is natural in origin, below.
due to photochemical reactions in the A. 1 only
troposphere.
B. 2 and 3
C. 1 and 3
45. Consider the following statements with
respect to Nitrate poisoning: D. All of the above.

1. Methemoglobinemia is caused by the Answer: - D


decreased ability of blood to carry vital Explanation:
oxygen around the body.
All of the above are the result of coal
2. One of the most common causes is combustion at thermal power plants.
nitrate in drinking water. Pollution from Coal-Based Thermal Power
Which of the statements given above is/are Stations
correct? • Coal-based thermal power stations
A. 1 only contribute over half sulphur dioxide
(SO2), 30% oxides of nitrogen (NOx),
B. 2 only
about 20% particulate matter (PM), etc.
C. Both 1 and 2 Persistent burning of coal in thermal
D. None of the above. power stations and a delay in
implementation of latest carbon
Answer: - C
capture storage technology are among
Explanation: major reasons of air pollution in India.
• Methemoglobinemia is caused by the
decreased ability of blood to carry vital
47. How is the National Green Tribunal
oxygen around the body. One of the
(NGT) different from the Central Pollution
most common causes is nitrate in
Control Board (CPCB)?
drinking water. It is most important in
bottle-fed infants and water from wells 1. The NGT has been established by an
in rural areas is of special concern. Act whereas the CPCB has been
created by the executive order of the
government.
46. Which of the following is/are emitted
2. The NGT provides environmental
from coal combustion at thermal power
justice and helps reduce the burden of
plants – (PYQ)
litigation in the higher courts whereas
1. Carbon Dioxide the CPCB promotes cleanliness of
2. Oxides of Nitrogen streams and wells, and aims to
improve the quality of air in the
3. Oxides of Sulphur
country.
Which of the statements given above is/are B. Use of plants to remove contaminants from
correct? (2018) soil and water.
A. 1 only C. Process of enhancing the degradation
B. 2 only process with the help of light.

C. Both 1 and 2 D. Processing of contaminated solid material


or water through an engineered
D. Neither 1 nor 2
containment system.
Answer: - B
Answer: - B
Explanation:
Explanation:
• The National Green Tribunal has been Statement B is correct.
established on 18 October, 2010 under
the National Green Tribunal Act 2010 Additional Information
for effective and expeditious disposal • Phytoremediation is a bioremediation
of cases relating to environmental process that uses various types of
protection and conservation of forests plants to remove, transfer, stabilize,
and other natural resources including and/or destroy contaminants in the
enforcement of any legal right relating soil and groundwater.
to environment and giving relief and There are several different types of
compensation for damages to persons phytoremediation mechanisms. These are:
and property and for matters
• Rhizosphere biodegradation: In this
connected therewith or incidental
process, the plant releases natural
thereto.
substances through its roots,
• The Central Pollution Control Board supplying nutrients to
(CPCB) of India is a statutory microorganisms in the soil. The
organisation under the Ministry of microorganisms enhance biological
Environment, Forest and Climate degradation.
Change (MoEFCC). It was established
• Phyto-stabilization: In this process,
in 1974 under the Water (Prevention
chemical compounds produced by the
and Control of Pollution) Act, 1974. A
plant immobilize contaminants, rather
function of CPCB comes under both
than degrade them.
national level and as State Boards for
the Union Territories. • Phyto-accumulation (also called
phyto-extraction): In this process,
plant roots absorb the contaminants
48. Phytoremediation refers to the along with other nutrients and water.
A. Process of decomposition of organic matter The contaminant mass is not destroyed
present in water by microorganisms using but ends up in the plant shoots and
oxygen. leaves. This method is used primarily
for wastes containing metals.
• Phyto-volatilization: In this process, • The processes involved in removing
plants take up water containing the contaminants include physical
organic contaminants and release the processes such as settling and
contaminants into the air through their filtration, chemical processes such as
leaves. disinfection and coagulation and
biological processes such as slow sand
• Phyto-degradation: In this process,
filtration.
plants actually metabolize and destroy
contaminants within plant tissues. Coagulation / Flocculation
• Aluminium sulphate (alum) is the
49. Which of the following are the methods most common coagulant used for
of Sewage Water Treatment for Domestic water purification.
Use? • Other chemicals, such as ferric
1. Coagulation sulphate or sodium aluminate, may
also be used.
2. Disinfection
• During coagulation, liquid aluminium
3. Fluoridation
sulfate (alum) is added to untreated
4. Sedimentation water.
Select the correct answer using the code given • This causes the tiny particles of dirt in
below: the water to stick together or
A. 1 only coagulate.

B. 2 and 4 only • Next, groups of dirt particles stick


together to form larger particles called
C. 3 only
flocs.
D. 1, 2, 3 and 4
• Flocs are easier to remove by settling or
Answer: - D filtration.
Explanation: Sedimentation
• Coagulation, Disinfection, • As the water and the floc particles
Fluoridation and Sedimentation are progress through the treatment
methods of sewage Water Treatment process, they move into sedimentation
for Domestic Use. basins where the water moves slowly,
Additional Info causing the heavy floc particles to
settle to the bottom.
Sewage Water Treatment for Domestic Use
• Floc which collects on the bottom of
• Substances that are removed during
the basin is called sludge and is piped
the process of drinking water
to drying lagoons.
treatment include suspended solids,
bacteria, algae, viruses, fungi, and
minerals such as iron and manganese.
• In Direct Filtration, the sedimentation However, since ozone is unstable, it
step is not included, and the floc is cannot be stored and must be
removed by filtration only. produced on-site, making the process
Filtration more expensive than chlorination.

• Water flows through a filter designed • Ozone has the advantage of not
to remove particles in the water. The causing taste or odour problems. It also
filters are made of layers of sand and leaves no residue in the disinfected
gravel, and in some cases, crushed water.
anthracite. • The lack of an ozone residue, however,
• Filtration collects the suspended makes it difficult to monitor its
impurities in water and enhances the continued effectiveness as water flows
effectiveness of disinfection. The filters through the distribution system.
are routinely cleaned by backwashing. Fluoridation
Disinfection • Water fluoridation is the treatment of
• Water is disinfected before it enters the community water supplies for the
distribution system to ensure that any purpose of adjusting the concentration
disease-causing bacteria, viruses, and of the free fluoride ion to the optimum
parasites are destroyed. level sufficient to reduce dental caries.

• Chlorine is used because it is a very • Fluoride is generally present in all


effective disinfectant, and residual natural water. Its concentration up to a
concentrations can be maintained to certain level is not harmful. Beyond
guard against possible biological that level, the bones start
contamination in the water disintegrating. This disease is called
distribution system. fluorosis.

• The addition of chlorine or chlorine • We have fluoride problem in many


compounds to drinking water is called parts of our country. Bureau of Indian
chlorination. Standards prescribes 1.0 mg/l as
desirable and 1.5 mg/l as a maximum
• Chlorine can combine with certain permissible limit for drinking water.
naturally occurring organic
compounds in water to produce • Defluoridation at domestic level can be
chloroform and other potentially carried out by mixing water for
harmful by-products. treatment with an adequate amount of
aluminium sulphate (alum) solution,
• The risk of this is very small, however, lime or sodium carbonate and
when chlorine is applied after bleaching powder depending upon its
coagulation, sedimentation, and alkalinity (concentration of
filtration. bicarbonates and carbonates in water)
• Ozone gas may also be used for and fluoride contents.
disinfection of drinking water.
systems; and urban storm water runoff
50. Consider the following statements: etc.

1. Biological Oxygen Demand is the amount o Higher BOD indicates more oxygen is
of dissolved oxygen needed by bacteria in required, signifying lower water
decomposing the organic wastes present in quality. Low BOD means less oxygen is
water. being removed from water, so the
water is usually more pure.
2. Chemical Oxygen Demand is the amount
of oxygen required to oxidize organic and • Chemical Oxygen Demand (COD) is the
oxidizable inorganic compounds in the amount of oxygen that is consumed when
water sample. a water sample is chemically oxidised.

Which of the statements given above is/are • The difference between BOD and COD is:
correct? COD or Chemical Oxygen Demand is the
total measurement of all chemicals
A. 1 only
(organics & in-organics) in the water,
B. 2 only whereas, BOD is a measure of, the amount
C. Both 1 and 2 of oxygen that require for the bacteria to
degrade the organic components present
D. None of the above.
in water.
Answer: - C
Explanation:
51. Consider the following statements with
Both the statements are correct. respect to Microplastics:
• Biochemical Oxygen Demand (BOD) 1. These are plastic pieces of fiber, which are
o It is the amount of oxygen that is very small, generally measuring less than
dissolved and consumed by biological 1mm.
organisms when they decompose 2. They have the ability to spread easily and
organic matters in water bodies. provide silky texture and colors to the
o Biological oxygen demand is product.
essentially a measure of the amount of 3. They are non-biodegradable and flow
oxygen required to remove waste through sewers to seas and oceans.
organic matter from water in the
Which of the statements given above is/are
process of decomposition by aerobic
correct?
bacteria.
A. 1 and 2
o Main sources of BOD are: leaves and
woody debris; dead plants and B. 2 and 3
animals; animal manure; effluents C. 1 and 3
from pulp and paper mills, wastewater
D. All of the above.
treatment plants, feedlots, and food-
processing plants; failing septic Answer: - D
Explanation: expressing the relative intensity of sound on a
scale from 0 to 130.
• Microplastics are plastic particles less than
5mm in diameter. • Noise is unwanted sound and has become a
part of urban life and industrial centres in this
• Primary Microplastics: They are tiny
century. Noise pollution may come from
particles designed for commercial use and
loudspeakers, factories, aeroplanes, moving
microfibers shed from clothing and other
trains, construction activity or even a radio.
textiles. E.g. microbeads found in personal
care products, plastic pellets and plastic • Noise level of 80 decibels or more for more than
fibres. 8 hours a day increases tension and changes in
breathing patterns. Continued exposure to
• Secondary Microplastics: They are formed
high levels of noise results in fatigue, hearing
from the breakdown of larger plastics such
loss or even total loss of hearing, changes in
as water bottles.
blood circulation, changes in breathing, etc.
• Noise pollution above 120 decibels can cause
52. Which of the following methods can be
many adverse biochemical changes.
used to control noise pollution?
Cholesterol levels in the blood and white cell
1. To enforce acoustic zoning by distancing counts increase, besides causing hypertension.
human settlements from industrial areas,
aerodromes and railway stations.
53. Consider the following statements with
2. Silence zones should be created for respect to Indicator species:
educational institutes, hospitals and
1. Lichens Indicate high oxygen
important offices.
content in water.
3. Sound-proof chambers should be installed
2. Stoneflies indicate low air pollution.
for the machines generating loud noise.
Select the correct code: 3. Mosses indicate acidic soils.

A. 1 and 2 only Which of the statements given above is/are


correct?
B. 2 and 3 only
A. 1 only
C. 1 and 3 only
B. 2 only
D. 1, 2 and 3
C. 3 only
Answer: - D
D. 2 and 3
Explanation:
Answer: - C
All the statements are correct.
Explanation:
• Noise pollution is a serious threat to the quality
Statement 1 and 2 are interchanged.
of man's environment. Noise, by definition, is
over-loud or disturbing sound.Sound levels are • Lichens – indicate low air pollution
measured in decibels (dB). It is a unit for
• Stoneflies –Indicate high oxygen content
in water. • Anthropogenic causes include industrial
emissions, burning of fossil fuels such as diesel
• Mosses – indicate acidic soils.
and coal, incineration of garbage, production of
paper.
54. Consider the following statements with
• Natural causes could be release of sulfur
respect to these toxic gases:
during volcanic eruptions or nitrogen ions
1. Sulfur dioxide inhibits photosynthesis by released in the atmosphere during a lightning
disrupting the photosynthetic mechanism. strike. The chemical reaction occurs in the
2. NOx gases react to form smog and acid rain presence of lightning to form the nitric oxide.
as well as being central to the formation of This further reacts with oxygen to form
tropospheric ozone. nitrogen dioxide.

3. Ozone may cause inhibition of CO2 • Furthermore, ozone, some other organic acids
acquisition when water supplies are like formic and acetic acids also contribute to 5-
adequate, 20% acidity in total acid rain.

Which of the statements given above is/are


correct? 55.Consider the following with reference to
A. 1 and 2 Biomagnification:

B. 2 and 3 1. It can be defined as the rise or increase in


the contaminated substances caused by the
C. 1 and 3
intoxicating environment.
D. All of the above.
2. Pollutant must be soluble in fats and
Answer: - D biologically active.
Explanation: 3. Bio-magnification makes humans more
• All are correct. prone to cancer, kidney problems, and liver
failure.
Acid Rain
Which of the above statement is/are correct?
• Acid rain is the rain that has been acidified,
with a pH less than 5.6. A. 1 and 2 only

• Excessive amounts of sulfur and nitrogen B. 2 and 3 only


released by cars and industrial processes get C. 1, 2 and 3
mixed with rain and result in precipitation that
D. 1 and 3 only
is highly acidic.
Answer: - C
• These pollutants react with water vapours
Explanation:
present in the atmosphere to form sulfuric acid
and nitric acid respectively. All of the above statements are correct.

• Sulfur and Nitrogen particles may be released • Bio-magnification can be defined as the
in the atmosphere due to anthropogenic causes rise or increase in the contaminated
or by natural causes. substances caused by the intoxicating
environment. The contaminants might be consultation with IIT-Kanpur and an expert
heavy metals such as mercury, arsenic, group comprising medical and air-quality
and pesticides such as polychlorinated professionals.
biphenyls and DDT.
• These substances are taken up by the
organisms through the food they
consume. When the organisms in the
higher food chain feed on the organisms in
the lower food chain containing these
toxins, these toxins get accumulated in the
higher organisms.

56. Which of the following are correct with


respect to National Air Quality Index?
57. Consider the following points about
1. It was launched by the Environment
National Clean Air Programme (NCAP):
Ministry under the Swachh Bharat
initiative. 1. It is a short-term, time-bound programme
to eliminate air pollution.
2. It helps the common man to judge the air
quality within his vicinity. 2. It aims at 20% to 30% reduction of PM2.5
and PM10 concentration levels by 2024,
3. It has been developed by the National
with 2020 as the base for concentration
Green Tribunal.
levels.
Select the correct answer using the code given
3. State-specific plans are part of the
below.
programme.
A. 1 and 2
Which of the above statements is/are not correct?
B. 2 and 3
A. 1 and 2 only
C. 1 and 3
B. 2 and 3 only
D. All of the above.
C. 1 and 3 only
Answer: - A
D. 1, 2 and 3
Explanation:
Answer: - D
• The measurement of air quality is based on
Explanation:
eight pollutants, namely: Particulate Matter
(PM10), Particulate Matter (PM2.5), Nitrogen • Statement 1 is incorrect: The Ministry of
Dioxide (NO2), Sulphur Dioxide (SO2), Carbon Environment, Forest and Climate Change
Monoxide (CO), Ozone (O3), Ammonia (NH3), launched the NCAP in January 2019. NCAP
and Lead (Pb). aims to:

• It has been developed by the CPCB in o prepare comprehensive mitigation actions


for prevention, control and abatement of air 3. At present vast majority of coal-based
pollution, power plants do not possess this
technology.
o augment the air quality monitoring
network and strengthen awareness Which of the above statements is/are correct?
activities. It is a long-term, time-bound, A. 1 and 3 only
national level strategy to tackle air
pollution problem across the country in a B. 2 and 3 only
comprehensive manner with targets to C. 1, 2 and 3
achieve 20% to 30% reduction in Particulate
D. 1 and 2 only
Matter concentrations (PM 2.5 and PM 10)
by 2024 (with 2017 as base year). Answer: - C
Explanation:
• Statement 2 is incorrect: (NCAP) is a long-term,
time-bound, national level strategy to tackle air • Statement 1 is correct as it a set of
pollution problem across the country in a technologies used to remove Sulphur
comprehensive manner with targets to achieve dioxide from fossil-fuel power plants.
20% to 30% reduction in Particulate Matter
• Statement 2 is correct as Flue gas
concentrations (PM 2.5 and PM 10) by 2024
Desulphurization can use various
(with 2017 as base year).
methods like wet scrubbers, spray dry
• Statement 3 is incorrect: City-specific action system or dry injection system.
plans are to be formulated for the 102
• Statement 3 is correct as at present.
nonattainment cities identified by CPCB. The
city action plans have been prepared to control
specific air pollution sources through 59. Consider the following:
multidimensional actions by bringing several
1. Bio-sparging—is a process of stimulating
implementation agencies together. Expansion
the natural in situ biodegradation of
of ambient air quality network, source
contaminants in soil by providing air or
apportionment studies, public awareness,
oxygen to existing soil microorganisms.
grievance redressal mechanism and sector
specific action points are part of these action 2. Bio-venting—is an in-situ remediation
plans. technology that uses indigenous
microorganisms to biodegrade organic
constituents in the saturated zone.
58. Consider the following with reference to
Which of the above statement(s) is/are correct?
Flue Gas Desulphurization:
A. 1 only
1. It is a set of technologies used to remove
Sulphur dioxide from fossil-fuel power B. 2 only
plants. C. Both 1 and 2
2. It can use various methods like wet D. Neither 1 nor 2
scrubbers, spray dry system or dry
Answer: - D
injection system.
Explanation: • Most of these are synthetic organics, which are
Both the statements are incorrect as they are of high concern.
interchanged. • Some naturally occurring organic compounds
• Bio-venting is a process of stimulating the biodegrade so slowly that they are considered
natural in situ biodegradation of contaminants refractory.
in soil by providing air or oxygen to existing • For example, lignic acid, tannic acid, cellulose,
soil microorganisms. Bio-Venting uses low air and many of the organics associated with
flow rates to provide only enough oxygen to petroleum.
sustain microbial activity in the vadose zone.
• Pesticides (DDT, aldrin, dieldrin, endrin,
• Bio-sparging is an in-situ remediation endosulfan, lindane, toxaphene, mirex,
technology that uses indigenous heptachlor etc.), polychlorinated biphenyls
microorganisms to biodegrade organic (PCB’s), polyaromatic hydrocarbons (PAH’s),
constituents in the saturated zone. In plastics, dioxins, furans, phenols, dyes,
biosparging, air (or oxygen) and nutrients (if detergents etc.
needed) are injected into the saturated zone to
Additional Info
increase the biological activity of the
indigenous microorganisms Chemical pollutants can further be divided into
Organic pollutants and Inorganic pollutants.
Organic pollutants
60. Which of the following is/are non-
• Organic pollutants are the compounds that
biodegradable organic pollutants?
contain carbon atoms in them and mainly of
1. DDT biological origin.
2. Plastics • These are the oxygen depleting substances in
3. Aldehydes water. They are carried to the water course
from different sources like sewage, animal and
4. Detergents
human excreta, refineries,
Select the correct answer using the code given
• distillery, pulp and paper industry, tanneries,
below:
dairy industry, textile industry, slaughter
A. 1, 2 and 4 only houses etc.
B. 2 and 3 only • BOD and COD is a common indicator of high
C. 3 and 4 only organic content of wastewater.

D. 1, 2, 3 and 4 • The organic pollutants could be further


subdivided into two.
Answer: A
Biodegradable organic pollutants
Explanation
• These are the organic materials which can be
Non-biodegradable organic pollutants
easily degraded by microorganisms.
• These are the organic materials which are
• Most of the naturally occurring organic
resistant to biological degradation.
compounds (originated from plants and • The Bharat Stage (BS) are emission standards
animals) are biodegradable. instituted by the Government of India to
regulate the output of air pollutants from
• Examples: Proteins, fats, carbohydrates,
motor vehicles.
alcohols, acids, aldehydes, esters, soaps etc.
Inorganic pollutants • The Environment Ministry is responsible for
deciding the fuel standard in the country. The
• Inorganic pollutants include inorganic acids, Central Pollution Control Board implements
alkalis, salts, anions, cations, free chlorine, these standards.
ammonia etc.
• The BS regulations are based on the European
• They are added as a result of industrial emission standards.
effluents, sewage, household cleansers and
surface run-off from urban and agricultural • Companies can come up with new vehicles
fields etc. with BS VI fuel standards even before the April
2020 deadline. But after the deadline, vehicles
• They affect the physical and chemical quality of that do not comply with BS VI standards will
water. not be registered.
• With the implementation of new norms,
61. Consider the following statements pollution levels are expected to come down
regarding difference Between BS-IV and significantly as the particulate matter (PM) will
BS-VI norms: decrease. About one-third of the air pollution is
1. In BS-VI fuel, the volume of PM 2.5 ranges caused by cars and motor vehicles.
from 20 to 40 micrograms per cubic metre Advantages of BS-VI fuel Over BS-IV Fuel
whereas in BS-IV fuel it is up to 120
• In BS-VI fuel, the volume of PM 2.5 ranges from
micrograms per cubic metre.
20 to 40 micrograms per cubic metre whereas
2. BS-VI fuel will bring down sulphur content in BS-IV fuel it is up to 120 micrograms per
by 5 times from the current BS-IV levels. cubic metre.
Which of the above is/are correct? • BS-VI fuel will bring down sulphur content by
A. 1 only 5 times from the current BS-IV levels. It has 10
ppm of sulphur as against 50 ppm in BS-IV.
B. 2 only
o Sulphur in the fuel contributes to fine
C. Both 1 and 2
particulate matter emissions. High sulphur
D. Neither 1 nor 2 content in the fuel also leads to corrosion
Answer: C and wear of the automobile engine.

Explanation • With BS-VI fuel, for every one kilometre, a car


will emit 80% less particulate matter and nearly
• Both the statements are correct.
70% less nitrogen oxide.
Additional Info
• Air pollutants in BS-VI fuel are much less as
Bharat Stage Emission Standards compared to BS-IV fuel.
• BS-VI norms also seek to reduce the level of o Increased growth
certain harmful hydrocarbons in the emissions o Primary production and biomass of algae
that are produced due to incomplete
combustion of fuel. o Changes in the balance of organisms
o Water quality degradation.

62. Which among the following statements • The consequences of eutrophication are
is/are correct in context to the effect of undesirable if they appreciably degrade
eutrophication on ecosystem: ecosystem health and biodiversity and/or the
sustainable provision of goods and services.
1. Increase in algal growth
Primary Impacts
2. Increase in dissolved oxygen
• Algal growth in marine waters is regulated by
3. Decrease in Biological oxygen demand
the level of nitrogen and phosphorous and to a
4. Shift in species composition lesser extent other inorganic compounds.
Select the correct answer code: • Eutrophication leads to an increased algal
A. 1, 2 and 3 only growth (because the level of nutrients
increases).
B. 2 and 3 only
• It can lead to a shift in species composition to
C. 2,3 and 4 only
fast growing algae species (including toxic
D. 1 and 4 only species) and a shift from long lived macroalgae
Answer: - D to more nuisance species.

Explanation: Secondary Impacts


• The reduced amount of sunlight received by
bottom waters can lead to the reduction in the
depth distribution of macroalgae and sea
grasses.
• Increased decomposition of organic matter
(dead algae) can lead to oxygen deficiency in
bottom waters. Lowered oxygen
concentrations can then impact the fish and
benthic fauna (animals living on the bottom of
the sea or a lake), which either flee or die from
the area.
What is Eutrophication?
• In the end, eutrophication can cause a shift in
• Eutrophication is a process driven by the the biodiversity and ecosystem balance.
enrichment of water by nutrients, especially
compounds of nitrogen and/or phosphorus,
leading to: 63. The Objective of Stockholm convention
is to protect human health from Persistent
Organic Pollutants (POPs). Which of the toxaphene
following statements is/are correct in context
• Industrial Chemicals:
to POPs?
o Hexachlorobenzene, polychlorinated
1. They are highly toxic.
biphenyls (PCBs)
2. They do not degrade into less toxic form.
• By products:
3. They can travel long distance through air &
o hexachlorobenzene; polychlorinated
water
dibenzo-p-dioxins and polychlorinated
Which of the statements given above is/are dibenzofurans (PCDD/PCDF), and PCBs.
correct
POPs
A. 1 and 2 only
• Persistent Organic Pollutants are a group of
B. 2 and 3 only toxic chemicals widely used as pesticides
C. 1 and 3 only and/or industrial chemicals.

D. All of the above These chemicals share four common properties:

Answer: - C • They are highly toxic

The Stockholm Convention • They are persistent, lasting for years or even
decades in the environment before degrading
• Stockholm Convention on Persistent Organic
into less toxic forms
Pollutants is an international environmental
treaty, signed in 2001 and effective from May • They travel long distances through the air
2004. and/or water

• POPs is a global treaty to protect human health • They accumulate in the fatty tissues of people
and the environment from the potentially and the wildlife.
harmful effects of POPs.
• Parties to the Convention will take measures to 64. Among the following which is the best
control/restrict the trade, domestic production description of a Carbon negative company?
and use of intentionally produced POPs, and
A. It removes as much carbon dioxide from
to reduce, and where possible to ultimately
the atmosphere as they contribute.
eliminate, the production and release
of unintentionally produced POPs by- B. It removes more CO2 from the atmosphere
products. than it contributes.

• The convention listed 12 POPs as ‘dirty dozen’. C. It releases more CO2 to the atmosphere
than it removes.
Initially, twelve POPs have been recognized as
causing adverse effects on humans and the D. It balance the whole amount of greenhouse
ecosystem and these can be placed in 3 categories: gas released and the amount removed from
the atmosphere.
• Pesticides:
Answer: - B
o aldrin, chlordane, DDT, dieldrin, endrin,
heptachlor, hexachlorobenzene, mirex, Explanation:
• Carbon neutral means that an entity removes Why can’t we throw all our trash into a volcano
as much carbon dioxide from the atmosphere and burn it up?
as they contribute. • It’s true that lava is hot enough to burn up
• Carbon-negative means that an entity removes some of our trash. When Kilauea erupted on
more CO2 from the atmosphere than it the Big island of Hawaii in 2018, the lava flows
contributes. were hotter than 2,000 degrees Fahrenheit
(1,100 Celsius).
• Carbon negative means the same thing as
“climate positive.” • That’s hotter than the surface of the planet
Venus and hot enough to melt many rocks. It’s
• Net-Zero carbon emissions mean that an
also as hot as waste incinerators, which usually
activity releases net-zero carbon emissions into
burn garbage at 1,800 to 2,200 F (1,000-1,200 C).
the atmosphere.
• But not all lavas are the same temperature.
• Net-Zero emissions balance the whole amount
of greenhouse gas (GHG) released and the Beyond temperature, there are other good reasons
amount removed from the atmosphere. not to burn our trash in volcanoes.
• First, although lava at 2,000 degrees F can melt
many materials in our trash — including food
65. Volcanoes might seem like nature’s
scraps, paper, plastics, glass and some
incinerators that can burn garbage. Why
metals — it’s not hot enough to melt many
don’t we throw all our trash into a volcano
other common materials, including steel,
and burn it up?
nickel and iron
1. The temperature of volcanoes is not as high
as waste incinerators. • Second, there aren’t many volcanoes on Earth
that have lava lakes, or bowl-like craters full of
2. Dumping trash into active lava lakes can lava, that we could dump trash into. Of all of
cause an explosion. the thousands of volcanoes on Earth, scientists
3. It will supress release of toxic gases from know of only eight with active lava lakes
volcano which leads to groundwater
• If rocks or other materials fall onto the surface
pollution.
of a lava lake, they will break the crust, disrupt
4. It’s not hot enough to melt many common the underlying lava and cause an explosion.
materials like steel, nickel and iron
• Finally, many indigenous communities view
Select the correct answer using codes: nearby volcanoes as sacred places.
A. 1 and 2 only • For example, Halema’uma’u crater at Kilauea
B. 2 and 3 only is considered the home of Pele, the native
Hawaiian goddess of fire and the area around
C. 2 and 4 only
the crater is sacred to native Hawaiians.
D. 1,2 and 3 only Throwing trash into volcanoes would be a
Answer: - C huge insult to those cultures.

Explanation:
66. Fluoride poisoning is a major concern in Health Organization standards.
in few regions of India, which among the
• The study found that the accumulation of
following step can provide appropriate
fluoride was higher in leafy and non-leafy
solution to it:
vegetables than in pulses and cereals from both
1. Promote use of ground water and restrict districts.
use of surface water
• The maximum accumulation was found in
2. Ban use of leafy vegetables and promote onion, while being minimum in rice.
use of pulses .
• It also found that children were the most
3. Promote fluoride free tooth pastes vulnerable to fluoride contamination due to
throughout India their low body weight.
Select the correct answer code: • Fluoride is not carcinogenic like arsenic. It
A. 1 and 2 only attacks the calcium in the body

B. 2 and 3 only Causes

C. 3 only • Contaminated water, therefore, is being used


to cultivate crops for years due to the lack of
D. None of the above
government intervention.
Answer-D
• The concentration of fluoride in groundwater
Explanation above the permissible level is, therefore, a
Fluoride has poisoned agricultural soil, crops in factor that contributes to the accumulation of
Bengal fluoride in agricultural soil and crops
• Recently new study has pointed out how crops There are three stages to fluoride toxicity:
and vegetables that use fluoride-contaminated • First, the groundwater used for agricultural
groundwater have been contributing to an purposes deposits a good amount of fluoride in
increase in consumption of fluoride among the soil.
locals.
• This fluoride is absorbed by crops
Key findings
• It enters the food chain system, causing harm
• West Bengal has been grappling with
to the human body.
groundwater contamination for decades.
Impact
• About 12 per cent of the population in eight of
the state’s 23 districts is impacted by water • Fluoride is an essential micronutrient and has
contaminated by fluoride. both beneficial and detrimental effects on
human health.
• The contamination is higher in western parts of
the state such as Purulia and Bankura. • However, exposure to high levels of fluoride
causes dental fluorosis, skeletal fluorosis and
• Groundwater in Purulia and Bankura is
non-skeletal fluorosis.
already known to be fluoride-contaminated, as
it crosses the desirable limit of 1.5 milligram • Ligaments calcification, liver and kidney
per litres of concentration as per the World dysfunction, nerve weakness, developmental
disorder, organ tissue damage, bending of legs, C. 3 only
weakness, anemia, depression, gastrointestinal D. All of the above
problems, loss of appetite, and brittle bone
problems in children are some of the Answer: A
commonly seen health issues Explanation
Steps Detergent footprint
• To address the fluoride menace, the West Impact of detergent
Bengal government introduced a Fluoride Task Nonylphenol, a hazardous chemical
Force Committee.
• It is present in detergents, is known to enter
• The committee, in collaboration with UNICEF, water bodies and the food chains.
has distributed filtering devices to households
• It bio-accumulates and can pose serious
in these districts. These devices filter at least 20
environmental and health risks.
liters of drinking water.
• It has been detected in human breast milk,
• Nano technology and electro-coagulation
blood and urine, and is associated with
methods can be used to filter out the
reproductive and developmental effects in
contaminated water.
rodents
• Surface water resources are important. We
Phosphate salts
need to focus on rainwater harvesting, so that
there is minimum dependence on groundwater • Many laundry detergents contain
approximately 35 to 75 per cent phosphate
• Water pipelines need to be installed in affected
salts.
areas to provide large scale water supply.
• Phosphates can cause a variety of water
pollution problems. For example,
67. Consider the following statements phosphate tends to inhibit the
regarding consequences of detergent biodegradation of organic substances.
footprint:
• Non-biodegradable substances cannot be
1. It bio-accumulates and can pose serious eliminated by public or private
environmental and health risks. wastewater treatment
2. Phosphate based detergents tends to • Some phosphate-based detergents can also
inhibit the biodegradation of organic cause eutrophication.
substances.
• Phosphate-enrichment can cause the water
3. High concentration of detergent kills fishes body to become choked with algae and
whereas fish eggs are unharmed due to other plants.
their solid layer.
• Eutrophication deprives the water of
Select the correct answer code:
available oxygen, causing the death of
A. 1 and 2 only other organisms.
B. 2 and 3 only
• In Belgium, phosphates are restricted for emissions of carbon monoxide and sulphur
use in household detergents since 2003. oxide.
Other Which of the above is/are correct?

• Detergents are capable of destroying the A. 1 only


external mucus layers that protect B. 2 and 3 only
the fish from bacteria and parasites,
C. 3 only
causing severe damage to the gills.
D. 1, 2 and 3
• Mostly fish diewhen detergent concentrat
ions are near 15 parts per million (ppm); Answer: D
however, detergent concentrations as low Explanation:
as 5 ppm will kill fish eggs
• Statement 1 is correct: It is derived from corn
• Turbidity also clogs the respiratory system and sugarcane using fermentation process.
of some species of fishes.
• Statement 2 is correct: A litre of ethanol
• Detergents also contain oxygen-reducing contains approximately two thirds of the
substances (ie, a chemical compound that energy provided by a litre of petrol.
readily transfer oxygen atoms) that may
• Statement 3 is correct: When mixed with petrol,
cause severe damage to the fishes and
it improves combustion performance and
other marine animals
lowers the emissions of carbon monoxide and
Standards sulphur oxide.
• The Bureau of Indian Statistics (BIS) has
set the standard of phenolic compounds in
69. Consider the following statements:
drinking water {0.001 milligram per litre
(mg/L)} and surface water (5.0 mg/L). 1. Methanol is a low carbon, hydrogen
carrier fuel produced from high ash
• The detergents contain suspected
coal
carcinogens, and ingredients that do not
fully biodegrade. 2. 'Methanol Economy' programme is
aimed at reducing India's oil import
bill
68. Consider the following statements with
3. Methanol is the best pathway for
reference to Bioethanol:
meeting India’s commitment to COP
1. It is derived from corn and sugarcane using 21.
fermentation process.
4. Di-methyl Ether, a derivative of
2. A litre of ethanol contains approximately methanol can be blended with LPG
two thirds of the energy provided by a litre
Which of the above is/are correct?
of petrol.
A. 1 and 2 only
3. When mixed with petrol, it improves
combustion performance and lowers the B. 3 and 4 only
C. 4 only bring down GHG emissions by 20% in terms of
particulate matter, NOx, and SOx, thereby
D. 1, 2, 3 and 4
improving the urban air quality.
Answer: D
• Methanol Economy will also create close to 5
Explanation:
million jobs through methanol
• Statement 1 is correct: Methanol is a low production/application and distribution
carbon, hydrogen carrier fuel produced from services. Additionally, Rs 6000 crore can be
high ash coal saved annually by blending 20% DME (Di-
methyl Ether, a derivative of methanol) in LPG.
• Statement 2 is correct:'Methanol Economy'
This will help the consumer in saving between
programme is aimed at reducing India's oil
Rs 50-100 per cylinder.
import bill
• Statement 3 is correct: Methanol is the best
pathway for meeting India’s commitment to 70. Which of the following is/are the Impact of
COP 21. E20 as a Fuel?

• Statement 4 is correct: Di-methyl Ether, a 1. The use of E20 as fuel reduces carbon
derivative of methanol can be blended with monoxide emission more for four
LPG wheelers than two wheelers

• Methanol is a low carbon, hydrogen carrier fuel 2. Hydrocarbon emissions also reduce
produced from high ash coal, agricultural compared to unblended petrol
residue, CO2 from thermal power plants and Which of the above is/are correct?
natural gas. It is the best pathway for meeting
A. 1 only
India’s commitment to COP 21.
B. 2 only
• NITI Aayog's 'Methanol Economy'
programme is aimed at reducing India's oil C. Both 1 and 2
import bill, greenhouse gas (GHG) emissions, D. Neither 1nor 2
and converting coal reserves and municipal
Answer: B
solid waste into methanol.
Explanation:
• Although slightly lower in energy content
• Statement 1 is incorrect: The use of E20 as fuel
• than petrol and diesel, methanol can replace
reduces carbon monoxide emissions by 50% in
both these fuels in the transport sector (road,
two-wheelers and 30% in four-wheeler
rail and marine), energy sector (comprising DG
vehicles.
sets, boilers, process heating modules, tractors
and commercial vehicles) and retail cooking • Statement 2 is correct: Hydrocarbon emissions
(replacing LPG [partially], kerosene and wood also reduce compared to unblended petrol
charcoal). • Impact of E20 as a Fuel
• Blending of 15% methanol in gasoline can Impact on Environment
result in at least 15% reduction in the import of
• The use of E20 as fuel reduces carbon
gasoline/crude oil. In addition, this would
monoxide emissions by 50% in two-wheelers Efficiency (NMEEE).
and 30% in four-wheeler vehicles. Answer: B
• Hydrocarbon emissions also reduce compared Explanation:
to unblended petrol.
• Statement 2 is incorrect: The PAT scheme is a
• Ethanol blending can thus reduce emissions in market-based mechanism wherein energy
vehicles. saving certificates are given as incentives to
Impact on Consumers industries that overachieve their targets. These
certificates can be traded in an energy exchange
• The fuel efficiency of vehicles will reduce by:
(Indian Energy Exchange or Power Exchange
• 6-7% for 4 wheelers designed for E0 and India). The certificates can be bought by other
calibrated for E10 DCs that have not achieved their prescribed
• 3-4% for 2 wheelers designed for E0 and targets under the scheme.
calibrated for E10 • Units that are unable to achieve the targets
• 1-2% for 4 wheelers designed for E10 and either by their own actions or by buying the
calibrated for E20 energy saving certificates are liable to pay the
prescribed penalty.
• However, with improvements in engines, the
loss in fuel efficiency can be minimised. PAT Scheme

Impact on Vehicle Manufacturer • PAT is a cyclic scheme where certain notified


energy intensive units having threshold energy
• Engines and components will need to be tested
consumption are given Specific Energy
and calibrated with E20 as fuel.
Consumption (SEC) reduction targets over a
• No major change in the assembly line is cycle of three years. PAT Scheme or the
required. Perform, Achieve and Trade Scheme was
launched by the Bureau of Energy Efficiency
• Vendors need to be developed for the
(BEE) in July 2012.
procurement of additional components
compatible with E20 o The chief goal of the scheme is to make
India’s industrial sector energy efficient.

71. Which of the statement is incorrect with o The scheme sets energy efficiency targets
reference to PAT Scheme? o for industries with those failing to achieve
A. PAT is a cyclic scheme the targets having to pay a penalty. The
penalty is based on what remains to be
B. The PAT scheme is not a market-based
achieved in terms of the target.
mechanism
o PAT scheme is a part of the BEE’s National
C. The chief goal of the scheme is to make
Mission for Enhanced Energy Efficiency
India’s industrial sector energy
(NMEEE).
efficient.
D. PAT scheme is a part of the BEE’s
National Mission for Enhanced Energy 72. Consider the following statements:
1. The objective of the Bio- fuel policy is to
achieve 20% ethanol-blending and 5% 73. With reference to Green Term Ahead Market,
biodiesel-blending by the year 2030. consider the following statements:
2. National Biofuel Coordination Committee 1. The renewable developers get into long
is headed by - Prime-Minister term PPAs
Which of the above is/are correct? 2. Transactions through GTAM will be
A. 1 only bilateral in nature
B. 2 only 3. GTAM contracts will be segregated into
Solar RPO & Non-Solar RPO
C. Both 1 and 2
Which of the statements given above is/are
D. Neither 1 nor 2
correct?
Answer: C
A. 1 only
Explanation:
B. 2 and 3 only
• Statement 1 is correct: The objective of the Bio-
C. 3 only
fuel policy is to achieve 20% ethanol-blending
and 5% biodiesel-blending by the year 2030. D. 1, 2 and 3

• Statement 2 is correct: National Biofuel Answer: D


Coordination Committee is headed by - Prime- Explanation:
Minister
• Statement 1 is correct: It is an alternative new
National Policy on Bio-fuels model introduced for selling off the power by
• The National Policy on Bio- fuels 2018 has the renewable developers in the open market
empowered the National Bio-fuel without getting into long term PPAs.
Coordination Committee (NBCC) to allow • Statement 2 is correct: Transactions through
conversion of surplus quantities of food-grains GTAM will be bilateral in nature with clear
for production of ethanol during an agriculture identification of corresponding buyers and
crop year when there is projected oversupply sellers, there will not be any difficulty in
of food-grains. accounting for Renewable Purchase
• The objective of the Bio- fuel policy is to Obligations (RPO).
achieve 20% ethanol-blending and 5% • Statement 3 is correct: GTAM contracts will be
biodiesel-blending by the year 2030. The policy segregated into Solar RPO & Non-Solar RPO
also expands the scope of feedstock for ethanol
production and has provided for incentives for
production of advanced bio-fuels. 74. Which of the following is/are the benefits of
Green Day Ahead Market?
• National Biofuel Coordination Committee is
headed by - Prime-Minister to provide policy 1. Deepen Green Market
guidance and coordination. 2. Accelerate the renewable capacity addition
3. Focus on PPAs power, unlock untapped renewable energy
Select the correct code: potential, ensure instant payment to
Renewable Energy generators i.e. on the day of
A. 1 only delivery itself.
B. 2 and 3 only
C. 3 only 75. Consider the following statements:
D. 1 and 2 only 1. Green hydrogen is produced by
Answer: D electrolysis of water using renewable
energy.
Explanation:
Expected Benefits: 2. Brown hydrogen is produced using coal
where the emissions are released into the
• Statement 1 is correct: Deepen Green Market: It air.
will deepen the green market and will provide
competitive price signals, besides offering an 3. Blue hydrogen is produced from natural
opportunity to the market participants to trade gas where the associated emissions are
in green energy, in the most transparent, released into the air.
flexible, competitive, and efficient manner. Which of the above statements is/are correct?
• Statement 2 is correct: Accelerate the A. 1 only
Renewable Capacity Addition: It will provide B. 2 and 3
another option to renewable generators to sell
C. 1 and 2
power as well as accelerate the renewable
capacity addition towards India's vision as a D. 1, 2 and 3
sustainable and efficient energy economy. The Answer: C
distribution utilities would also be able to sell
Explanation:
surplus renewable power generated in their
area. • Statement 1 is correct: Green hydrogen is
produced by electrolysis of water using
• Statement 3 is incorrect: Shift from PPA based
renewable energy (like Solar, Wind) and has a
Contract to Market-Based Models:
lower carbon footprint. Electricity splits water
o It will create a domino effect that will lead into hydrogen and oxygen. By Products:
to a gradual shift from Power Purchase Water, Water Vapor.
Agreements (PPAs) based contracts to
market-based models. • Statement 2 is correct: Brown hydrogen is
produced using coal where the emissions (By
o It will build and deepen the markets to the products) are released into the air.
next level, paving the way for India to meet
its ambitious target of 450 GW green • Statement 3 is incorrect: Blue hydrogen is
capacity by 2030. produced from natural gas, where the
emissions are captured using carbon capture
Reduction of Curtailment of Green Power:
and storage.
• It would reduce the curtailment of green
76. Which of the following is not an advantage of Observer Status to the International Solar
Electric Vehicles? Alliance (ISA).
A. Have the potential to reduce greenhouse What is the International Solar Alliance (ISA)?
gas emissions
• Co-founded by India and France during 2015,
B. Reduce dependence on petroleum the ISA is an action-oriented, member-driven,
C. Reduce health effects from air pollution collaborative platform for increased
deployment of solar energy technologies. Its
D. They produce tailpipe emissions
basic motive is to facilitate energy access, ensure
Answer: D energy security, and drive energy transition in
Explanation: its member countries. ISA is the nodal agency
for implementing One Sun One World One
• Statement 1 is correct: Have the potential to Grid (OSOWOG), which seeks to transfer solar
reduce greenhouse gas emissions power generated in one region to feed the
• Statement 2 is correct: Reduce dependence on electricity demands of others.
petroleum • The Headquarters is in India with its Interim
• Statement 3 is correct: Reduce health effects Secretariat being set up in Gurugram.
from air pollution • A total of 109 countries have signed the ISA
• Statement 4 is incorrect: They produce little or Framework Agreement and 90 have ratified it.
no tailpipe emissions • All member states of the United Nations are
eligible to join the ISA.
77. Which of the following statements is
incorrect with reference to International
78. Consider the following statements:
Solar Alliance? 1. The Global Solar Council (GSC) is an
A. It is co-founded by India and France international non-profit association of the
B. All member states of the United Nations are national, regional and international
eligible to join the ISA. associations in solar energy and the world.
C. The United Nations General Assembly 2. National Solar Energy Federation of India
(UNGA) has granted Observer Status to the (NSEFI) is an umbrella organization of all
International Solar Alliance (ISA). solar energy stakeholders of India
D. None of the above Which of the above is/are correct?
Answer: D A. 1 only
Explanation: B. 2 only
C. Both 1 and 2
• Statement 1 is correct: It is co-founded by India D. Neither 1 nor 2
and France
Answer: C
• Statement 2 is correct: All member states of the Explanation:
United Nations are eligible to join the ISA.
• Statement 1 is correct: The Global Solar Council
• Statement 3 is correct: The United Nations (GSC) is an international non-profit association
General Assembly (UNGA) has granted
of the national, regional and international anchored in the seabed.
associations in solar energy and the world.
• Floating wind farms are still in their infancy.
• Statement 2 is correct: National Solar Energy
• Offshore wind farms must be at least 200
Federation of India (NSEFI) is an umbrella
nautical miles from the shore and 50 feet deep
organization of all solar energy stakeholders of
in the ocean.
India
• Offshore wind turbines produce electricity
which is returned to shore through cables
79. Consider the following with reference to buried in the ocean floor.
Offshore Wind Energy:
• The coastal load centres distribute this
1. Offshore wind farms have alower
electricity based on priority.
capacity utilization factor (CUF) than
onshore wind farms. • In India, where land is limited and the
population is increasing, large wind farms
2. Offshore wind turbines require
positioned over water bodies will be vital.
stronger structures and foundations
than onshore wind farms.
Which of the above is/are correct? 80. Consider the following statement with
reference to National Wind-Solar Hybrid Policy:
A. 1 only 1. The Policy provides for the integration of
B. 2 only both energy sources i.e., wind and solar at
alternating current (AC) as well as direct
C. Both 1 and 2 current (DC) level.
D. Neither 1 nor 2 2. It seeks to promote new hybrid projects as
well as hybridisation of existing wind and
Answer: B solar projects.
Explanation: 3. It will be on the tariff-based transparent
bidding process for which Government
• Statement 1 is incorrect: Offshore wind farms entities may invite bids.
have a higher capacity utilisation factor (CUF) Which of the above is/are correct?
than onshore wind farms.
A. 1 and 2 only
• Statement 2 is correct: Offshore wind turbines B. 3 only
require stronger structures and foundations C. 2 and 3 only
than onshore wind farms. D. 1, 2 and 3

Offshore wind energy refers to the deployment of Answer: D


wind farms inside the water bodies. They utilise the Explanation:
sea winds to generate electricity. Features of National Wind-Solar Hybrid Policy
• These wind farms either use fixed-foundation Some of the important features of the National
turbines or floating wind turbines. Wind-Solar Policy are mentioned below:
• A fixed-foundation turbine is built in shallow • It has been provided in a hybrid project, subject
water, whereas a floating wind turbine is built to the condition that, rated power capacity of
in deeper waters where its foundation is
one resource be at least 25% of the rated power A. 1 only
capacity of other resources for it to be B. 2 only
recognised hybrid project.
C. Both 1 and 2
• The Policy provides for the integration of both
energy sources i.e., wind and solar at D. Neither 1 nor 2
alternating current (AC) as well as direct Answer: C
current (DC) level. Explanation:
• It seeks to promote new hybrid projects as well • Statement 1 is correct: The amount of power
as hybridisation of existing wind and solar that can be harvested from wind depends on the
projects. It allows hybridisation of existing size of the turbine and the length of its blades.
projects (wind or solar) with higher The output is proportional to the dimensions of
transmission capacity than sanctioned one, the rotor and to the cube of the wind speed.
subject to availability of margin in existing
transmission capacity. o Theoretically, when wind speed doubles,
wind power potential increases by a factor
• It will be on the tariff-based transparent bidding of eight
process for which Government entities may
invite bids. • Statement 2 is correct: The National Offshore
wind energy policy has the objective to develop
• The policy permits the use of battery storage in the offshore wind energy in the Indian
hybrid projects for optimising output and Exclusive Economic Zone (EEZ) along the
reducing variability. Indian coastline
• It mandates the regulatory authorities to
formulate necessary standards and regulations 82. Consider the following statements:
for wind-solar hybrid systems. 1. Ministry of New and Renewable
Energy for large hydro projects and the
mandate for the small hydro power is
81. Consider the following statements with
reference to Wind energy: given to Ministry of Power.

1. The amount of power that can be 2. Small hydro is defined as any


harvested from wind depends on the hydropower project which has an
size of the turbine and the length of its installed capacity of less than 50 MW
blades. 3. India and China are the major players
2. The National Offshore wind energy of the SHP sector, holding the highest
policy has the objective to develop the number of installed projects.
offshore wind energy in the Indian Which of the above is/are correct?
Exclusive
A. 1 only
3. Economic Zone (EEZ) along the Indian B. 2 and 3 only
coastline C. 3 only
D. 1 and 2 only
Which of the above is/are correct?
Answer: C
Explanation: Statement 2 is incorrect: Tropical rainforest
soils are often deep but not particularly
• Statement 1 is incorrect: Ministry of Power,
fertile. This is partly due to the fact that huge
Government of India is responsible for large
amounts of some mineral nutrients are tied
hydro projects, the mandate for the subject
up inside the vegetation rather than being
small hydro power (up to 25 MW) is given to
free in the soil at any given time.
Ministry of New and Renewable Energy.
Statement 3 is correct: Tropical rain forests
• Statement 2 is incorrect: Small hydro is defined
help in maintaining the water cycle to protect
as any hydropower project which has an
against flood, drought, and erosion.
installed capacity of less than 25 MW
• Statement 3 is correct: India and China are the
major players of the SHP sector, holding the 84. Consider the following pairs of plants
highest number of installed projects. and their examples:

Plants Examples
83. Consider the following statement with
reference to Tropical Rain Forests:
1. Hydrophytic plants Water lily
1. Brazil, the Democratic Republic of
Congo , and Indonesia are home to the
world's largest rainforests. 2. Xerophytic plants Cactus

2. Tropical rainforest soils are often deep


and very fertile. 3. Mesophytic plants Corn

3. Tropical rain forests help in


Which of the above is/are correctly
maintaining the water cycle to protect matched?
against flood, drought, and erosion.
A. 1 and 2 only
Which of the above is/are correct?
B. 2 and 3 only
A. 1 and 2 only
C. 1 and 3 only
B. 2 and 3 only
D. 1,2 and 3
C. 1 and 3 only
Answer: D
D. 1, 2 and 3
Explanation
Answer: C
Based on their water requirements and
Explanation adaptations, plants are divided into three
Statement 1 is correct: Brazil (South major groups: (1) Hydrophytes, (2)
America), the Democratic Republic of Congo Mesophytes, (3) Xerophytes and.
(Africa), and Indonesia are home to the Statement 1 is correct: Hydrophytic plants,
world's largest rainforests. such as water lily or pondweed, grow in
saturated soil or water
Statement 2 is correct: Xerophytic plants, deciduous forest; dry forest; dry-deciduous
such as cactus, grow in extremely dry soil, forest; tropical deciduous forest.
mesophytes are ordinary plants that exist Mediterranean Biome is also known as
between the two extremes. Temperate Deciduous Biome
Statement 3 is correct: Mesophytic plants are Statement 4 is incorrect: Teak, neem,
adapted to neither too dry nor too wet type bamboos, sal, shisham, sandalwood, khair,
climate. A majority of plants living on this mulberry are some of the important species
planet are mesophytes. Example: corn, rose, of Tropical Monsoon Forests.
clover, squash, etc.

86. Consider the following statement with


85. Consider the following statement with reference to Tropical Grasslands:
reference to Mediterranean Climate: 1. They have cold winters and warm
1. Trees with small broad leaves are summers
widely spaced and never very tall. 2. African savanna is one of the tropical
2. Fire is an important hazardous factor grasslands.
in this ecosystem 3. It is an excellent hunting ground.
3. It is also known as drought-deciduous Which of the above is/are correct?
forest
A. 1 only
4. Teak and neem are some of the
important species. B. 2 and 3 only

Which of the above is/are correct? C. 2 only

A. 1 and 2 only D. 1 and 3 only

B. 3 and 4 only Answer: B

C. 1 and 3 only Statement 1 is incorrect: Tropical grasslands


have dry and wet seasons that remain warm
D. 2 and 4 only all the time. These regions are warm
Answer: A throughout the year, followed by dry and
Explanation wet seasons. Tropical Grasslands are the ones
which receive 50 cm to 130 cm rain.
Statement 1 is correct: Trees with small broad Temperate Grasslands are similar to tropical
leaves are widely spaced and never very tall. grasslands, except for the climatic conditions.
Statement 2 is correct: Fire is an important They have cold winters and warm summers
hazardous factor in this ecosystem, and the with 25 cm and 75 cm. of annual rainfall.
adaptation of the plants enable them to Shrublands are the best example of temperate
regenerate quickly after being burnt. grasslands.
Statement 3 is incorrect: Tropical Monsoon Statement 2 is correct: African savanna is one
Forests are also known as a drought- of the tropical grasslands.
Statement 3 is correct: The tropical grasslands ● The forest also has diverse climatic
contain quite short plants which makes it an patterns and different species of
excellent hunting ground. organisms.

87. Consider the following statement with Statement 1 is correct: This biome is only
reference to Taiga Biome: located in Northern Hemispheres at latitudes
1. This biome is only located in Northern between 60° and 50° North. The forest
Hemispheres stretches in a belt pattern across the Northern
hemisphere covering approximately 100
2. It is mainly made up of coniferous
million acres of land. Due to the ability of the
trees
plants found in this forest to survive in
3. Fungi such as mushrooms are found snowy or extremely cold temperatures, the
in large quantities here. location of the boreal forestcan be seen to be
Which of the above is/are correct? between the temperate deciduous forests that
are located on the South Side and the Tundra
A. 1 and 2 only
located on the North.
B. 2 and 3 only
Statement 2 is correct: Boreal forest is mainly
C. 1 and 3 only made up of coniferous trees evergreens such
D. 1,2 and 3 as the spruces, pines, and larches. These trees
maintain the green color to ensure that they
Answer: D utilize the minimum sunlight available and
Explanation to start the process of photosynthesis early.
● The boreal forest is the planet’s The conifers have characteristic needle-like
second-largest biomestretching in leaves that are waxy that ensure that they lose
unbroken pattern across Europe, Asia, a minimal amount of water during the
and North America and covers an summer and spring periods.
extensive 20 million hectares. Statement 3 is correct: Fungi such as
● The forest is also referred to as Taigaor mushrooms are found in large quantity in the
the snow forest. boreal forest. The fungi are saprophytes and
get enough nutrients from the many dead
● It is mainly made up of coniferous branches and trees found within the forest.
forestand trees such as larches, The common fungi found at the root of the
spruces, birches, aspens, firs, and conifers are the mycorrhizal fungi.
pines.
● The tree coverage in most parts of the
88. Consider the following statements with
forest is extensive and close forming a
reference to Arctic Tundra Biome:
canopy with moss undergrowth.
1. It is considered a major carbon sink
● But in some areas, the trees are apart
with an undergrowth of lichens 2. The arctic tundra biome is the
referred to as lichen woodland. northernmost biome.
3. Plants in this biome tend to go humus. Peat is decayed sphagnum moss and
dormant during the long winters. humus is organic matter.
Which of the above is/are correct? Statement 2 is correct: The arctic tundra
A. 1 only biome is the northernmost biome. It covers
the lands north of the Arctic Circle up to the
B. 2 and 3 only polar ice cap. It reaches as far south as the
C. 3 only Hudson Bay area of Canada and the northern
part of Iceland. It covers approximately 11.5
D. 1,2 and 3
million km2.
Answer: D
Statement 3 is correct: Plants in this biome
Explanation also tend to go dormant during the long
● The arctic tundra is a vast, dry, rocky winters. This means that they slow down
place that is noted for its lack of trees. their normal life functions.
● In fact, the word “tundra” comes from
the Finnish word tunturi, meaning 89. The Cactus plant has many features
‘treeless plain.’ One important which helps it to thrive in harsh desert
characteristic of the tundra is the biome. Which of these features is/are
permafrost. correct?
● The word permafrost is short-form for 1. Shallow roots
the permanently frozen soil, which
2. Spongy interior
starts within a meter of the soil
surface. In the winter almost all of the 3. Slow growth
soil is frozen. Select the correct code:
● In the summer the soil near the surface A. 1 and 2 only
thaws, but the permafrost at lower
B. 2 and 3 only
depths remain frozen.
C. 1 and 3 only
● The permafrost limits how far roots of
plants can extend down into the soil. D. 1,2 and 3

● It also is what prevents trees from Answer: D


growing. Explanation
● The ground in the arctic tundra tends ● The cactus plant, particularly, has
to be rocky and the soil has few adapted well to the harsh climatic
nutrients. conditions of desert biomes such that
Statement 1 is correct: Despite the lack of it’s able to grow up to 20 feet or more
trees, this biome is still considered a major and liver for over 200 years.
carbon sink as there are large amounts of ● The Giant Saguaro cactus, in
organic matter found in deposits of peat and particular, has managed to thrive in
this harsh biome due to shallow roots,
which enable it to soak up rain and hardly inhabited, and their average
dew before it evaporates. It also population density is lower than one
features a spongy interior, which has person per square kilometer. Of all the
the ability to expand and hold a lot of deserts in the world, Australian deserts
water. Another adaptation of the have the lowest human population, by
cactus is that it grows slowly to save far.
energy. Afrotropic Deserts
• These desert biomes are located in Sub-
90. Consider the following statement with Saharan Africa, including the southern
reference to Desert Biome: fringe of the Arabian Peninsula. The
1. In Desert Biome the rate of biome receives a lot of pressure from
evaporation typically overtakes humans, particularly in Madagascar and
precipitation. the horn of Africa.

2. The desert biomes of the world are The Indo-Malay region


located in six biogeographic domains • The Indo-Malay region consists of 2 hot
3. The largest hot desert biome is the lowlands including the Indus Valley and
subtropical Sahara. the Thar. These deserts top the world
deserts regarding human footprint.
4. Fox are unable to survive in the Desert
Biome. The Neotropic Deserts

Which of the above is/are correct? • Those located in South America cover an
area of about 684, 000 million square
A. 2 and 3 only
miles. However, only 6 percent of this
B. 1 and 4 only total area is protected.
C. 1, 2 and 3 only The Nearctic Deserts
D. 2,3 and 4 only • These deserts cover an estimated area of
Answer: C 1.04 million square miles in North
America. Due to the expansion of urban
Explanation
corporations like Phoenix in the U.S., their
Statement 1 is correct: In Desert Biome the average population is relatively high.
rate of evaporation typically overtakes
Palearctic Domain
precipitation.
• This domain concentrates the largest
Statement 2 is correct: The desert biomes of
range of desserts in the entire world,
the world are located in six biogeographic
covering an area of a staggering 9.9
domains including:
million square miles. That’s, essentially,
The Australian Deserts 63% of all deserts in the world. These
• The Australian deserts consist of a deserts are famous for their extreme
collection of lowlands arid eco-regions in dryness and sheer inaccessibility. The
the heartland of Australia. They are Sahara desert in Africa covers an area of
about 9.9 square miles or approximately This ratio could change, however, due to
10% of the African continent. On the flip variations in the sun's characteristics from
side, the deserts located in the Central time to time, thus leading to changes to the
area are characterized by folded global energy balance above and hence
mountains, high landscape heterogeneity, changes in our climate
as well as enclosed basins. Option b is correct: Variations in the earth's
Statement 3 is correct: The largest hot desert orbit around the sun changes in the tilt of the
biome (the subtropical Sahara) stretches over earth's axis, changes in the shape of the earth's
3.5 million square miles or 9 million square orbit (i.e. eccentricity) and the shifting of the
kilometers. equinoxes (i.e. precession) play major roles in
determining variations in the amount of sun's
Statement 4 is incorrect: Foxes survive here
energy reaching the earth and hence lead to
due to their burrowing and nocturnal
variations in the earth's climate (i.e. Climate
lifestyles. This burrowing ensures they escape
Change).
the intense heat during the day. They also
have large ears to aid in the dissipation of
excess body heat during hot days. Their thick, 92. Which of the following UV Radiation is
sandy fur helps protect them from the cold responsible for the immediate tanning?
nights in the desert. The thick, sandy fur also
A. UV-A
helps reflect heat and for camouflage.
B. UV-B
C. UV-C
91. Which of the following is not an external
factor of climate change? D. All of the above
A. Solar variability Answer: A
B. Earth’s orbit alteration Explanation:
C. Variability within the climate system About UV-A:

D. Volcanic activity • Relatively long-wavelength.

Answer: C • Accounts for approximately 95% of the


UV radiation reaching the Earth's
Explanation:
surface.
Climate change may be due to internal
• Penetrate into the deeper layers of the
processes and/or external forces. The
External Climate Forcing Processes are: skin and is responsible for the
immediate tanning.
Option a is correct: Variations in the amount
of energy received from the sun. About 30- • Enhances the development of skin
40% of the UV energy from the sun is believed cancers.
to be reflected back into space after hitting the
earth's upper atmosphere, while the
93. Consider the following statements regarding
remaining 60- 70% gets through to the earth. SAFAR:
1. It is an initiative of the Ministry of operationalized by the India
Environment, Forest, and Climate Change. Meteorological Department (IMD).It
has a giant true color LED display that
2. It is an integral part of India’s first Air
gives out real-time air quality index on
Quality Early Warning System operational
a 24x7 basis with color-coding (along
in Delhi.
with 72 hours advance forecast).
3. Ammonia is one of the pollutants that it
• The ultimate objective of the project is
monitors.
to increase awareness among the
Which of the statements given above is/are general public regarding the air quality
correct? in their city so that appropriate
A. 1 only mitigation measures and systematic
B. 2 and 3 only action can be taken up. It organizes
awareness drive by educating the
C. 2 only public (prompting self-mitigation),
D. 1 and 3 only and
Answer: - C • It also helps the policy-makers to
develop mitigation strategies keeping
Explanation:
in mind the nation’s economic
Statement 1 is incorrect: The System of Air development.
Quality and Weather Forecasting And
Research (SAFAR) is a national initiative • SAFAR is an integral part of India’s
introduced by the Ministry of Earth Sciences first Air Quality Early Warning System
(MoES). operational in Delhi. It monitors all
weather parameters like temperature,
Statement 3 is incorrect: Pollutants rainfall, humidity, wind speed, and
monitored: PM2.5, PM10, Ozone, Carbon wind direction, UV radiation, and
Monoxide (CO), Nitrogen Oxides (NOx), solar radiation.
Sulfur Dioxide (SO2), Benzene, Toluene,
Xylene, and Mercury. • Pollutants monitored: PM2.5, PM10,
Ozone, Carbon Monoxide (CO),
• The System of Air Quality and Nitrogen Oxides (NOx), Sulfur
Weather Forecasting And Research Dioxide (SO2), Benzene, Toluene,
(SAFAR) is a national initiative Xylene, and Mercury.
introduced by the Ministry of Earth
Sciences (MoES) to measure the air • The World Meteorological
quality of a metropolitan city, by Organization has recognized SAFAR
measuring the overall pollution level as a prototype activity on the basis of
and the location-specific air quality of the high-quality control and standards
the city. maintained in its implementation.

• The system is indigenously developed • SAFAR system would benefit cost


by the Indian Institute of Tropical savings to several other sectors like
Meteorology (IITM), Pune and is agriculture, aviation, infrastructure,
disaster management, tourism, etc. Act seeks to combat air pollution by
which directly or indirectly gets prohibiting the use of polluting fuels
affected by air quality and weather. and substances, as well as by
regulating appliances that give rise to
air pollution.
94. Consider the following statements
about The Air (Prevention and Control of • Statement 3 is correct: The Air Act
Pollution) Act, 1981 (the "Air Act"): empowers the State Government, after
consultation with the SPCBs, to declare
1. It provides for the establishment of
any area or areas within the Sate as air
Pollution Boards at the Central and State
pollution control area or areas.
levels with a view to prevent, control, and
abate air pollution.
2. It seeks to combat air pollution by
prohibiting the use of polluting fuels and
substances only.
3. It empowers the State Government, after A. Water Hyacinth
consultation with the State Pollution B. Hydrilla
Control Board, to declare any area or areas
C. Water Lily
within the State as air pollution control
areas or areas. D. All of the above.

Which of the statements given above is/are


correct?

Explanation:
• Statement 1 is correct: The Air
(Prevention and Control of Pollution)
Act, 1981 (the "Air Act") is an act to
provide for the prevention, control and
abatement of air pollution.
• Statement 2 is incorrect: To counter the
problems associated with air pollution,
ambient air quality standards were
established under the Air Act. The Air
100. Which of the following biome is also
called sclerophyll ecosystem or biome?
A. Tundra Biome
B. Taiga Biome
C. Temperate Deciduous Biome
D. Tropical Deciduous Biome
Answer: C
Explanation
The Mediterranean biome is also called as
sclerophyll ecosystem or biome because of the
develop-ment of special feature and
characteristic in the domi-nant trees and
shrubs to adapt to the typical environ-mental
conditions of the Mediterranean climates (dry
summer and wet winter).
● The Mediterranean climate has three
typical characteristic features which
give sclerophyll charac-teristics to the
vegetation of this biome :
○ Winters are cool but wet
whereas summers are dry, most
of the annual rainfall is received
during wet winter months
○ Summer season is warm and
dry whereas winters are
moderately cool
○ There is sufficient sunlight
throughout the year but
summer is more sunny.

You might also like